Endocrinology - workbook

Ace your homework & exams now with Quizwiz!

THYROID NEOPLASMS Benign thyroid adenomas

Follicular types (most common type) act autonomously causes hyperthyroidisms Rarely malignant Often non-functional

What antiviral used in treatment of CMV and HSV drops serum calcium?

Foscarnet

GFR? Medulla?

Glomerulosa = salt Fasciculata = sugar Reticularis = sex Medulla = stress

Treatment of hyperthyroidism with methimazole or PTU may cause what?

Goiter

Symptoms of Graves

Goiter (swelling of the neck): Excessive TSH receptor activation, parenchymatous hypertrophy and follicular hyperplasia causes symmetrical enlargement of the thyroid. Goiter is usually in hypothyroidism Exophthalmia: Curious eyeball protrusion and periorbital edema. The cause of exophthalmos is unknown; it may be related to autoimmune dysfunction and fibroblastic development in the affected eye. In pregnancy Grave's antibodies can cross the placenta and stimulate fetal rapid heart rate and slowed growth. At times they may block thyroid hormones and may cause cretinism in the infant. Lab Findings: Increased T3 and T4 and decreased plasma TSH due to T3 negative feedback inhibition Treatment of Graves - Propylthiouracil (PTU) and Methimazole: Inhibit peroxidase reaction and formation of iodine from iodide (stop iodination) and reduce T3 and T4 formation. Side effect: Increase TSH -> follicular stimulation => goiter PTU is short acting and requires several daily dosage whereas Methimazole requires one daily dosage - Use of 131I to destroy the thyroid - Surgical removal of the thyroid -beta blockers (propranolol) as adjunct therapy to alleviate generalized sympathetic effect of thyroid hormones

Goiter in Hypothyroidism and Grave's

Goiter is the cardinal characteristic of hypothyroidism, but Graves' (a hyperthyroid condition) also causes Goiter Hypothyroidism => Low T3 and T4 => Increased TSH => Follicular stimulation => Goiter Graves => Sensitized T-cells => Stimulation B-cells => TSI (thyroid antibody) => follicular stimulation => Goiter

COX-1?

Good COX Most bodily cells: stomach, intestine, kidney, etc. Involved in maintenance of various organs. Inhibition by classic NSAIDs and since you block - you may have GI toxicity, renal dysfunction, decreased platelet aggregation.

Wegener's Granulomatosis Also Causes eosinophilia. What are some of the main characteristics of this disease

Granulomatous inflammation of arterioles, capillaries and venules. Triad of Wegener's: (1) Focal lung vasculitis; (2) Necrotizing pulmonary granuloma; and (3) Necrotizing glomerulitis. Fifth decade onset. 80% of patients die in year one if not treated Immunosuppression (with cyclophosphamide) has saved more than 90% of the victims. C-ANCA (antineutrophilic cytoplasmic autoantibodies) is present in more than 90% of patients. It is a good disease marker

Squamous cell carcinoma of the lung

Grows fast and but mets later Produces PTH Tx: Surgically remove the lobes

Demography of Hypothyroidism?

Happens at any age. In infants may not show sx until 2-3 months. In the elderly sx may be confused with Parkinson's, Alzheimer's or depression.

A48-year-old diabetic female is admitted to hospital for complete examination to identify the cause of a swelling in her neck, and hypothermia of several-month duration. She has a puffy face, dry skin and coarse hair. Lab results confirm low serum T3 and T4, high TSH and detectable anti-thyroglobulin antibodies. These finding are most likely an indication of which of the following conditions?

Hashimoto's Thyroiditis Associated with other AI diseases: SLE, pernicious anemia, Sjogren's, DM, and RA Characterized by goiter patients mainly hypothyroid; a small proportion may be euthyroid

Wht are the top two causes of hypothyroidism in adults?

Hashimoto's and Iatrogenic (treatment of hyperthyroidism)

Type II DM

•AKA (old) non-insulin-dependent or NIDDM •Associated with insulin resistance, relative insulin deficiency and hyperglycemia •Primarily affects adults over age 40 and associated with obesity and sedentary lifestyle •Fewer tendencies for ketoacidosis •Possible complications: Nonketonic hyperglycemia, function impairment of many multiple organs including renal and cardiovascular system •Etiology: Genetics, medications such as steroids, hemochromatosis, and pancreatic diseases •Associations: Obesity, hypertension, metabolic syndrome, Acromegaly, cushing's disease, high-fat diet and sedentary life. •Diagnosis: Single raised glucose reading with symptoms or serum glucose values over 126 mg/dl on two samplings, or with glucose tolerance, plasma glucose over 200 mg/dl two hours after the oral glucose. •Treatment - Initially diet and exercise, and weight loss - Maintaining the HBA1C at the range of 6-7% - Self monitoring of blood glucose - Antidiabetic drugs: Metformin (a Biguanide), Sulphonylureas, and thiazolidinediones (TZDs) such as nateglinide, alpha-glucosidase inhibitors such as acarbose and miglitol - If Antidiabetic medications fail, insulin therapy may be required

Salicylates Aspirin, Salicylic Acid and Diflunisal

•Absorption via stomach and duodenum; elimination by conjugation via renal •Antipyretic effect; Non-selective inhibition of COX 1 and 2 enzymes (by acetylation); hence decrease PG production. Inhibition by Aspirin and not others is irreversible - new COX's synthesis needed to recover the aspirin effects •Decreases TXA-2; Decreases aggregation •Side-effects: Acidosis, uricosemia (with low doses), keratolysis (with Salicylates), gastric bleeding •Toxicity: Hypersensitivity; tinnitus, Reye's (in children with influenza or chickenpox)

Dx of Diabetes Type II

•Blood work - Random BG > 200 - Fasting BG > 110 - HbA1c has no role in diagnosis - Proteinuria •Exam - Obesity - Acanthosis nigricans - Retinopathy - Nonhealing ulcers

Physiology of Epinephrine

•Derivative of the amino acid tyrosine •Binds beta and alpha receptors •Physiologic actions - Heart: Positive chronotropy, inotropy, lusitropy, dromotropy - Vasculature: vasodilation (beta), vasoconstriction (alpha) - Kidneys: Increases renin secretion - Intestines: Decreases peristalsis and increases sphincter tone - Airways: Promotes bronchodilation - Brain: Euphoria

What is the pathogenesis?

Hereditary predisposition an/or association with past viral diseases such as measles or RSV Starts as ostoclastic and osteolytic activities followed by compensatory osteoblastic or mixed osteoclastic and osteoblastic activities, and ultimately ends with dense, deformed, large, and fragile bones mainly in spine, skull, pelvis, jaws, and legs sx: bone pain and enlargement, OA, and hearing loss

The definitive treatment for myxedema coma is IV levothyroxine. What is the most untoward drawback of high IV levothyroxine therapy?

High IV L-thyroxine may precipitate arrhythmias or myocardial infarction

Treatment Of Hypercalcemia

Hydration and saline diuresis with loop diuretics - Loop diuretics reduce serum calcium - Thiazides are contraindicated because they raise plasma calcium Bisphosphonates - Etidronate(oral or IV) and pamidronate - Inhibit osteoclastic activity - Indication: Paget's disease and osteoporosis Calcitonin: Human or salmon derived Glucocorticoids: Decrease intestinal absorption of calcium Plicamycin - Cytotoxic antibiotic - Inhibits PTH effect on osteoclasts

Calcitonin

Hydrophilic polypeptide made by parafollicular (C-cells) of the thyroid gland. Signal transduction: G protein kinase Positive Releasing Factors: Increased [Ca++] Negative Releasing Factors: Decreased [Ca++] Functions: Overall decreases plasma [Ca++] to prevent hypercalcemia and excess effect of PTH

Characteristics of hyperparathyroidism

Hypercalcemia causes kidney stone formation (common symptom) Hypophosphatemia Increased renal phosphate and decreased calcium excretion Increased urinary cAMP from kidney Increased bone resorption (dissolution) Increased fatigability

What are the top 2 immediate stimulants of aldosterone secretion?

Hyperkalemia and Angiotensin II

How do you distinguish 11 from 21?

Hypertension is the top most important distinguishing characteristic between 11 and 21-hydroxylase deficiencies. At the same time, the most common deficiency among all congenital adrenal deficiencies is the 21-hydroxylase deficiency. But the most commonly remembered one on the history of the exam (i.e. the most commonly tested one) has been the 17-hydroxylase deficiency!

Toxic Nodular Goiter (Plummer's Disease)

Hyperthyroid condition The second common cause of hyperthyroidism (after Grave's) Not all adenomatous nodules in the thyroid release thyroxine. In Plummer's some of the nodules are source of high thyroxine production (so called hot nodules). Occurs in elderly patients with pre-existing goiter Lack of circulating thyroid stimulators (as opposed to Grave's) Lack of ophthalmopathy (as opposed to Grave's) Lack of dermatopathy Contrast with Grave's: Generally less severe than Grave's except for the cardiovascular symptoms.

Functions of Thyroid Stimulating Hormone

Hypertrophy of follicles Increases iodide transport into the follicles Increases organification of tyrosine with iodide Increases coupling of mono-iodotyrosine and di-iodothyronine molecules, and thyroglobulin synthesis (T3 and T4) Increased secretion of T4 and to a lesser degree T3

After occluding blood flow in the arm with a sphygmomanometer a patient is presented with carpal spasm. What is the problem with this patient

Hypocalcemia - Trousseau sign

What are the top two side effects of mineralocorticoids?

Hypokalemia CHF due to volume overload

Riedel's (Fibrous) Thyroiditis

Hypothyroid state wherein the thyroid is replaced by hard fibrous tissues. Presented with goiter Fibrous tissues may extend to the neck

In which of the two conditions, use of glucocorticoids is more helpful; hyper or hypocalcemia?

In hypercalcemic situations! Large dose of glucocorticoids antagonize effects of active Vit D, and slow absorption of GI calcium and increase renal secretion of calcium.

What causes hypercalcemia?

In outpatients, the most common cause is hyperparathyroidism. In hospitalized patients, the most common cause is malignancy. The first test to order is the PTH, which helps differentiate hyperparathyroidism (high PTH) from other causes of hypercalcemia such as malignancy, vitamin D intoxication, or thiazide diuretic use (low PTH). Multiple types of cancer can cause hypercalcemia, but the classic USMLE question involves either multiple myeloma or secretion of PTH-like hormone by a squamous cell carcinoma, especially in the lung. Familial hypocalciuric hypercalcemia is characterized by hypercalcemia with low calcium levels in the urine (in contrast to other hypocalcemia). Other causes include vitamin A or D intoxication, sarcoidosis or other granulomatous diseases, and excessive calcium intake (milk-alkali syndrome).

What is the difference between a primary and secondary endocrine disorder?

In primary disorders, the problem is in the gland; the hypothalamic-pituitary axis is functioning appropriately. In primary hypothyroidism, for example, the thyroid gland does not function properly for whatever reason, but the pituitary and hypothalamus respond appropriately. Therefore, thyroid hormone is low (as in all cases of hypothyroidism), but TSH and thyroid-releasing hormone (TRH) are high (the appropriate response from the pituitary and hypothalamus to low levels of thyroid hormone). In secondary disorders, the true dysfunction is outside the gland itself. For example, in secondary hypothyroidism, thyroid hormone is low, but TSH and/or TRH is also low (inappropriate in the setting of low thyroid hormone). If the pituitary is destroyed or surgically removed, secondary hypothyroidism results from low TSH; the thyroid gland functions well, but no TSH is available to stimulate it. To confuse the picture, the dysfunction also may be completely outside the endocrine axis (e.g., heart failure that causes secondary hyperaldosteronism). This concept in endocrine gland dysfunction is quite important. Simple blood tests can localize the problem. You may be able to answer a USMLE question simply by reading through the various values for hormones and hormone-releasing factors and figuring out where in the hypothalamus-pituitary-target gland axis the problem lies.

Ectopic ACTH

Increase ACTH - primary cause Increase cortisol - follow No effect of dexamethasone effect

Leukotrienes?

Increase bronchial tone

Thromboxane

Increase platelet aggregation Increase vascular and bronchial tone

Positive TRH releasing factors?

Increased Calorie intake Decreased environmental temperature Increased Adrenergic input

METABOLIC EVENTS IN FAST AND FED STATES FASTING STATE ADIPOSE TISSUE

Increased FFA export Increased lipolysis Decreased FFA import Decreased glucose uptake

Negative CRH releasing factors?

Increased GABA Increased opioids Increased enkephalins

LIVER IN FASTING

Increased Glucose production and export Increased glycogenolysis Increased amino acid import from skeletal muscle Increased urea formation and export to kidney Increased gluconeogenesis Decreased glucose import Decreased glycolysis and glycogenesis Increased export of ketones Increased FFA import from adipose tissue Increased beta-oxidation and increased ketogenesis Decreased lipogenesis and triglyceride synthesis

Serum alk-P

Increased osteoblastic activity is associated with increased Alk-P Osteoblastic activity is high in children Increased osteoclastic increases serum Ca++

Negative TRH releasing factors?

Increased somatostatin from somatotrophs TSH will suppress hypothalamus Thyroxine will suppress Anterior Pituitary and Hypothalamus

CORTISOL EFFECTS ON THE LIVER

Increases amino acid (AA) uptake and gluconeogenesis. Increases glucose export (glucagon like!) Increases glycogen synthesis (insulin like!) Overall it has an anabolic effect in the liver

Ketosteroids and their significance

Ketosteroids are breakdown products of "male" steroid sex hormones (androgens) that would appear in plasma and urine. Normal Value of Urinary Ketosteroids - Males: 8 -20 mg/24 hours - Females: 6 -15 mg/24 hours Major Causes of Increased Urinary Ketosteroids: Adrenal cortical tumors, Cushing's, ovarian and testicular cancers, Polycystic Ovary Syndrome, Virilism and adrenal congenital hyperplasia (less common than the others) Major Causes of Decreased Ketosteroids: Addison's disease and hypopituitarism

Hepatic Disease: T3RU and TBG

Liver is an important site for T4 to T3 conversion and TBG synthesis. With liver disease initially T3 and later, T4 drops. T4 drop is primarily due to increased clearance than low production. T4 drop is accompanied by decreased TBG. Decreased TBG is associated with increased free T4. T3RU increases with increasing severity of disease Note that in biliary cirrhosis in contrast the level of TBG may increase

Eicosanoids

Locally synthesized and released as needed (de novo synthesis) Short half-life (one minute) Corticosteroids reduce all four eicosanoids; leukotrienes, prostacyclins, thromboxanes and prostaglandins Aspirin and NSAIDs inhibit the last three (they have no effect on lipoxygenase) PGE2(Prostin E2 or dinoprostone)and PGF2 (carboprost) increase uterine activity. Induce labor and abortion. Ibuprofen blocks PG synthesis and helps with dysmenorrhea Indomethacin blocks PG synthesis and closes PDA; PGI2 relaxes vascular; maintains PDA. Injected into penis induce erectile effects PGE bronchodilates and PGF bronchoconstriction PGE and PGI2 decrease gastric acid. (treat peptic ulcers) Misoprostol (a PGE1 derivative) reduces gastric ulceration from NSAIDs PGE and PGI2 sensitize afferent nerve endings to pain TXA2 of platelets vasoconstricts and PGI2 of vessel walls dilates IV infusion of Epoprostenol (a PGI2) treats pulmonary hypertension

What is the name of the condition that is associated with eosinophilia, fibrotic thickening of heart valves and restrictive cardiomyopathy

Loeffler's endocarditis (A.K.A. endomyocardial fibrosis) Loeffler's Obliterative Cardiomyopathy Endocarditis Fibrotic thickening of the heart Filling is reduced Latin (tropical) Eosinophilia Restrictive myopathy Shrunken cardiac cavities

At age 10 the patient is admitted to the hospital for complete physical evaluation. Which of the following lab findings would be the most remarkable finding in the patient

Low BP The karyotype of 46XX indicates that the patient has acquired congenital adrenal virilism (masculinization) as a result of too much androgens or over activity of zona reticularis. But by far 21-hydroxylase deficiency is the most common cause of congenital adrenal hyperplasia, and between the two options it seems to be the more probable answer

What are the top 2 serious side effects of the COX 2 inhibitors?

MI, thrombosis, stroke SJS (esp people with sulfa sensitivity)

Somatostatin

Made by parvocellular neurons of hypothalamus and delta-cells (delta cells) of pancreas Positive Releasing Factors: Increased level of glucose, FFA, GH, glucagon, and somatomedin Major Site of Action and Functions - Anterior Pituitary: Decreases growth hormone and TSH secretion - Pancreas:Decreases glucagon and Insulin secretion - GI Tract:Decreases GI hormones (gastrin, and CCK). Decreases gastric acid and pepsin secretion. Decreases blood flow, motility and CHO absorption. Increases water/electrolyte absorption Signal Transduction:Gi protein A kinase

Ovarian Androgen Secretion

Main ovarian androgens are androstenedione and testosterone. The theca externa and the interstitial cells synthesize the androgens. After menopause and decline of oocytes and follicles, continued secretion of androgens from the interstitium accounts for masculine characteristics such as hair growth.

What's significant about maternal thyroxine versus free thyroxine?

Maternal thyroxine is mostly joined to TBH and albumin and hardly passes the placenta. But free thyroxine will pass through placenta.

A forty-year-old woman with a history of tuberculosis is being evaluated for increasing weakness, anorexia and hypotension. Physical examination reveals tachycardia, and pigmentation of oral mucosa and skin. Lab examination is significant for hyponatremia, hyperkalemia, hypoglycemia, increased BUN, and decreased cortisol level. Plasma creatinine level is within normal range. Which of the following would be the most desirable diagnostic procedure in this case

Measurement of plasma cortisol level subsequent to ACTH infusion Failure of adrenals to increase cortisol after IV administration of ACTH is diagnostic of adrenal insufficiency. Note: Increased basal level or increased elevation of 17-hydroxyprogesterone after ACTH infusion is diagnostic for 21-hydroxylase deficiency. Diseases that destroy adrenals--TB, histoplasmosis, coccidioidomycosis, autoimmune diseases, etc are also associated with Addison's

4 types of Hypercortisolism Iatrogenic

Medication-induced; low ACTH Exogenous glucocorticoid therapy

What are some useful medications for hyperparathyroidism?

Medications include estrogen replacement therapy in postmenopausal women and bisphosphonates that increase the bone turnover.

Acromegaly

Most common cause is adenoma of the somatotropic cells Acromegaly in adults (after closure of long bones' epiphyses) Gigantism in children (less common than acromegaly) Cause: Increased GH release Features - Enlargement of head, hands, feet, jaw, and soft tissues—heart, liver, tongue, etc. - Bitemporal hemianopsia due to compression of optic chiasm - Usually changes happen gradually over 10 or more years GH producing cells are acidophilic (eosinophilic) 1/3 of GH producing tumors also produce Prolactin => infertility Treatment: Hypophysectomy plus hormone replacement therap

Prolactinomas

Most common functional pituitary tumor Hypogonadism in males and females; Galactorrhea and amenorrhea In females Increased prolactin suppresses GnRH Low GnRH leads to reduced LH and estradiol. This is the mechanism of infertility during pregnancy and nursing period. Causes: Prolactin-secreting cell tumors are less common. More common causes are: Hypothalamic disorders; Drugs impairing dopaminergic transmission; Estrogen therapy and high estrogen (raises prolactin and causes prolactinoma by unknown mechanisms via suppressing the hypothalamus and inhibiting dopamine (PIF) that in turn raises prolactin. Dopamine administration inhibits estrogen-induced prolactinoma

What is the relationship between myxedema and coma?

Myxedema coma is a misnomer because most patients exhibit neither the nonpitting edema (known as myxedema) nor coma

Addison's disease causes eosinophilia and darkened mucosa. What are the major causes of eosinophilia

NAACP is the mnemonic for the major causes of eosinophilia - Neoplasms (Hodgkin's and Leukemia) - Allergy (rhinitis, asthma, and eczema) - Addison's - Collagen diseases (WC, PAN, Pemphigus vulgaris) - Parasites/Pulmonary eosinophilia Note: Eosinophilia is often associated with skin hypersensitivity reactions that color the skin. In Addison's, ACTH is high and as such MSH is high as well.

NSAIDs and Gastritis

NSAIDs inhibit prostaglandin synthesis Prostaglandins are required for mucus layer synthesis and integrity. They also inhibit acid secretion. NSAIDs remove prostaglandin inhibition over acid secretion in the stomach (via cAMP) pH Changes and Aspirin - At low stomach pH, aspirin is uncharged and readily enters the mucosal cells - Within the mucosal cells it is ionized and becomes negatively charged. - Charged aspirin is trapped within the cells and will directly damage the mucosal cells

Suppression of renin in this patient is due to which of the following options?

Negative aldosterone feedback

This bug is one the memorable bugs of the exam. It has an antiphagocytic capsule that produces a lipopolysaccharide endotoxin. It causes meningitis in children after 6 months of age. Ceftriaxone is the most widely used DOC for it. One of the peculiar problems with this bug is that it may cause septic shock in some of its victims by flipping on a DIC reaction in the body. The consequence may be serious hemorrhagic damage to adrenals that would produce, among other things, an acute Addison disease-like syndrome. Who is this bug and what is the name of the complication that it produces in the adrenals

Neisseria meningitidis. The name of its complication is "Waterhouse-Friderichsen"

What causes nephrogenic DI?

Nephrogenic DI is caused by kidney unresponsiveness to ADH. Look for medications (e.g., lithium, demeclocycline) as the cause.

What is one the top etiologies of excess catecholamines in hypertensive children?

Neuroblastoma

Can you administer spironolactone to a patient with addison's disease?

No! Spironolactone requires a high aldosterone level in order to be effective. Triamterene and amiloride work better under hypoaldosteronism conditions such as Addison

Does aspirin directly have any anti-inflammatory, antipyretic and analgesic functions?

No. Aspirin is rapidly deactivated by esterases in the body, producing salicylic acid, which has all the above attribute

What is the normal serum value of HbA1c?

Normal serum value of HbA1 that indicates the degree of glycosylation of hemoglobin is about 6%

Type 2 will do the what?

Opposite of type 1

Tx of hyperparathyroidism

Oral phosphate Removal of adenoma

What is the name of bone-related disease that results from kidney failure?

Osteitis fibrosa cystica

SKELETAL MUSCLES IN FED STATE

Overall: Increased Protein Synthesis Increased glucose import Increased glycolysis and glycogenesis Decreased proteolysis Increased amino acid import

ADIPOSE TISSUE IN FED STATE

Overall: Increased TG synthesis Increased glucose uptake Increased glycolysis Increased glycerol synthesis Increased lipogenesis Decreased FFA exports to the liver Increased glucose uptake

Vit D Functions

Overall: Increases plasma [Ca++] and [PO4]. Provides substrate for bone formation Bone - Synergizes with PTH - Increases osteoclastic number and activity and decreases osteoblastic activity. Increases resorption and increases plasma Ca++level Kidney - Increases reabsorption and decreases excretion of Ca++ - Increases reabsorption and decreases excretion of PO4 Small Intestine - Increases [Calbindin] that transports Ca++across epithelial mucosal membranes and increases [Ca++] reabsorption from small intestine. - Increases PO4 reabsorption in the small intestine.

PTH FUNCTION

Overall: Increases plasma [Ca++] and decreases [PO4]. Prevents hypocalcemia and hyperphosphatemia Bone - Increases osteoclastic number and activity (increases resorption) - Decreases osteoblastic activity Kidney - Increases 1,25 dihydroxy-D3 formation - Decreases Ca++excretion by increasing Ca++reabsorption - Increases PO4 excretion by decreasing PO4 reabsorption Small Intestine - Indirectly via stimulating 1,25 dihydroxy-D3 (vitamin D) increases Ca++absorption

Physiologic stimuli => COX-1 constitutive => ?

PGE2 (kidney, mucal synthesis) TxA2 (platelet degranulation - exposed collagen to damaged endothelial walls) Prostacycline PGI2 (GI - suppress acid formation)

Prostacyclin?

PGI2 Decrease platelet aggregation Decreases vascular, uterine and bronchial tone

A 65 yo patient with childhood hx of measles and 6 month hx of joint and bone pain is admitted due to recent loss of hearing. Serum analysis indicates elevated Alk-P, normal calcium and phosphorus. Dx?

Paget's disease of bone (osteitis deformans)

Summary of Anti-hypercalcemics

Pamidronate and Etidronate Ethacrynic acid and furosemide Plicamycin Calcitonin Prednisone (glucocorticoids)

PSAMM mnemnoic

Papillary thyroid carcinoma Serous ovary cystadenocarcinoma Adenocarcinoma of ovary Meningioma Mesothelioma

Ectopic

Paraneoplastic Cushing's High ACTH Bilateral Adrenal Hypertrophy

Pheochromocytoma Pathophysiology, Rule of 10s, Genetics

Pathophysiology •Hypersecretion of epinephrine, norepinephrine, and dopamine => explaining variable presentation •Initiation of beta blocker may exacerbate hypertension due to unopposed alpha receptor mediated vasoconstriction Rule of 10's: •10% inherited •10% extra-adrenal •10% malignant •10% children •10% calcification •10% bilateral Genetics •Most pheochromocytomas arise sporadically •Approximately 10% associated with hereditary disorders such as MEN-2A or MEN-2B, von Hippel-Lindau (VHL) disease, or neurofibromatosis

Effects on Bone

Pathophysiology •Inhibits bone-forming cells (osteoblasts) •Stimulates bone-degrading cells (osteoclasts) •Reduces intestinal absorption of calcium and as a result, increased PTH-mediated bone resorption •Compromise blood supply to susceptible bones leads to avascular necrosis (e.g., hip) Prevention •DEXA scan •Supplemental calcium & vitamin D •Bisphosphonates

Grave's Disease

Pathophysiology •Stimulatory IgG Abs bind TSH receptors •Abs mimic TSH Diagnosis •Ophthalmopathy specific •Anti-TSH Abs specific Classic Presentation •Hyperthyroidism •Goiter •Ophthalmopathy •Dermopathy (pretibial myxedema)

A newborn presents with ambiguous external genitalia. The genitalia seem more like an enlarged clitoris than penis. There is a scrotum-like structure that has resulted from labial fusion. Ultrasound confirms normal ovarian development and presence of bilaterally enlarged adrenal glands. Karyotyping indicates a 46 XX karyotype. Of the following conditions which one is the most likely cause of these findings

Patients with 21 hydroxylase deficiency characteristically are hypertensive as a result of lack of development of zona glomerulosa or production of mineralocorticoids. Note that as result of low aldosterone and blood volume and pressure, the renin level is going to be high in these patients. Also as a result of lack of cortisol, the level of ACTH is also high in the patient. Also note that ketosteroids are the byproducts of metabolism of androgens such as testosterone or dihydrotestosterone. The patients with 21-hydroxylase deficiency have a very high level of androgens that raises the levels of ketosteroids in their serum and urine

Patients with Cushing's syndrome present with what? Prednisone is one of the what?

Patients with Cushing's syndrome present with shrunken lymphoid mass. Prednisone (a glucocorticosteroid) is one of the commonly used drugs in treatment of Hodgkin's lymphoma

Why myxedema coma at times is misdiagnosed as MI?

Patients with myxedema coma often have elevated Creatine Kinase and for this reason they may be misdiagnosed with myocardial infarction. Also, myocardial infarction happens to be the trigger for myxedema com

Metabolic Syndrome

Per ATP III ( > 3 of the following abnormalities): •Insulin resistance •Serum glucose level of at least 110 mg/dL •Abdominal obesity •Waist circumference > 102 cm in men and 88 cm in women •Hypertriglyceridemia •triglycerides > 150 mg/dL •Low HDL: Less than 40 mg/dL in men and 50 in women •HTN (hypertension) •Blood pressure of at least 130/85 mm Hg

Ps of Pheochromocytoma

Pheochromocytoma Postural/ Paroxysmal hypertension Premenopausal years (40-60) Palpitation Paragangliomas (10% outside adrenal) Polymodal Population Distribution (4-12 yr old boys and 40-60 yr women) Perspiration, Pallor Phenoxybenzamine Phentolamine

Growth Hormone (Somatotropin)

Polypeptide made by somatotrops of the anterior pituitary and released in pulsatile manner. Positive Releasing Factors: Growth hormone releasing hormone and wake/sleep transition Negative releasing Factors: Increased somatostatin, somatomedins, obesity, pregnancy and hyperglycemia Transduction:via Tyrosinase Kinase Actions - Very anabolic, crucial for normal growth - All Tissues: Increase lean mass, and decrease Adipose mass Liver: Synergizes with somatomedin to increase protein synthesis and gluconeogenesis Muscle: Increases AA and decreases glucose uptake. Increases protein synthesis Adipose: Increases lipolysis and decreases glucose uptake

What is exophthalmia?

Pop-eye or protruded eye common finding in grave's disease Etiology is related to growth of fibroblasts and GAGs at the back of orbit Postulated that immunoglobulins that cause grave's stimulate the fibroblast Hashimoto's as well and Grave's!

Sheehan's Syndrome

Postpartum infarction of the anterior pituitary gland - Pituitary enlarges during pregnancy. - Superimposed low-pressure portal blood or hypovolemia may predispose to ischemia May also occur in males and non-pregnant women—e.g. in DIC and vascular accidents Features: - Decreased FSH, LH, ACTH and TSH with decreased target hormones (e.g. cortisol) - Amenorrhea Treatment: Hormone replacement

What's the mechanism of aspirin in etiology of Reye's syndrome?

Postulated that it may be due to toxic interaction of salicylate on mitochondria that have been sensitized by viral infection Mitochondrial damage causes defect in fatty acid oxidation, elevated fatty acids, hyperammonemia (urea cycle failure in the liver) and cerebral edema.

Vitamin D (1,25 Dihydroxy-D3)

Precursors: D2 from diet and D3 from exposure of skin to UV. Conversion of inactive to active form happens in the kidney. Positive Releasing Factors - Increased PTH - PTH promotes activation of 1 alpha-hydroxylase which synthesizes active form of vitamin D Negative Releasing Factors - Decreased PTH and increased phosphate (phosphate inhibits 1 alpha hydroxylase) - Increased 1,25 OH2-D3 (negative feedback) Transduction at Targets: Via nuclear receptors which increase or decrease gene transcription

Why pressors and inotropes should be avoided in myxedema coma patients during L-thyroxine therapy?

Pressors and inotropes have the tendency to provoke arrhythmias in the setting of intravenous thyroid replacement

Low Renin

Primary Hyperaldosteronism (Conn's Syndrome) Cushing's Disease Congenital Adrenal Hyperplasia (11 and 17-hydroxylase deficiencies) Hyperthyroidism, Pheochromocytoma

ADDISON'S DISEASE

Primary adrenocortical insufficiency Causes - Autoimmune destruction (80%); and - Tuberculosis (20%); also Waterhouse Friderichsen due to N. meningitidis Demography: More in females ages 30-50 (female to male ratio is 2.5:1) Hormonal Characteristics - Insufficient adrenal glucocorticoids, androgens and mineralocorticoids. - Increased POMC and ACTH due to removal of negative feedback inhibition on the anterior pituitary. Clinical Correlations - Decreased cortisol causes hypoglycemia. - Increased POMC causes hyper-pigmentation. Decrease in aldosterone level causes decrease in Na+ and increase in K+retention, plus a decrease urinary acid secretion. Volume contraction, hypotension, hyperkalemia and metabolic acidosis Increased BUN as a result of volume contraction. Others Findings: Weakness, fatigue, GI disturbance, and electrolyte abnormalities

DIABETES INSIPIDUS

Primary: Deficiency of ADH Causes Surgical damage to posterior pituitary; Head trauma; Anoxia;Sheehan Syndrome (ischemic damage due pregnancy) Secondary (Nephrogenic): Insensitivity of nephrons to ADH. Symptoms of Diabetes insipidus - Volume loss and hypernatremia - Low urine specific gravity and osmolarity - Lack of ability to concentrate urine Treatment of Diabetes insipidus - Primary (intranasal desmopressin) Nephrogenic (thiazide diuretics)

Primary, Secondary, and Tertiary Hyperthyroidism

Primary: TRH is low, TSH low, T4 high Secondary: TRH low, TSH high, T4 high Tertiary: TRH high, TSH high, T4 high

Primary, Secondary and Tertiary Hypothyroidism

Primary: TRH up, TSH up, T4 is low Secondary: TRH up, TSH low, T4 is low Tertiary: TRH low, TSH low, T4 low

Inflammatory reactions => COX-2 inducible => ?

Pro-inflammatory PGs and Inflammatory mediators => Inflammation

GHRH

Produced in the hypothalamus Stimulates somatotrops of the anterior pituitary gland to secrete GH Positive Releasing Factors - Meal protein - Decreased glucose and FFA - Increased level of serotonin, alpha-adrenergic drive and dopamine - Fasting, exercise and stress Major site of action is anterior pituitary, where stimulates secretion of GH into the systemic circulation. Signal Transduction: Gs protein A kinase

Pregnenlone => Hydroxysteroid dehydrogenase =>?

Progeterone

What is the underlying mechanism for the antipyretic analgesic and anti-inflammatory functions of Aspirin?

Reduces PGs It causes analgesia in nerve endings, antipyresis in hypothalamus, and anti-inflammatory reaction at the reaction site.

SUBACUTE THYROIDITIS (THYROTOXICOSIS)

Release of preformed thyroxine from thyroid gland Commonly as a result of viral infections - painful!!! Often with interaction of HLA-B35 Starts with hyperthyroidism and turns into hypothyroidism and then gradually returns to normal. Thyroid function returns to normal after depletion of pre-made hormones. De Quervain's (granulomatous or painful sub-thyroiditis) is a subtype of subacute conditions.

High Renin

Renal Artery Stenosis Bartter's Syndrome (JG Tumor) Coarctation of Aorta (Adult Type) Nephrotic Syndrome Congestive heart failure, Abdominal artery stenosis

The two common side effects of salicylates, acetaminophen and phenylpropionic acid are GI bleeding and hepatorenal toxicity. Why do they cause renal toxicity?

Renal toxicity of salicylates and acetaminophen is due to reduction of PG in kidney. Reduced PG reduces renal flow and function. Resultant Salt and water retention may further effectiveness of medications

55 yo old man with a past hx of pulmonary TB is presented with sx of weight gain, fatigue, lethargy, and mental confusion. Lab findings include: low BUN, hypouricemia, decreased plasma osmolality; and increased urine osmolality. Dx?

SIADH Low BUN because you expand the ECF so dilution of BUN.

What type of eating behavior is not uncommon in Addison's?

Salt craving is the eating behavior that is common in Addison'

Metastatic cancer

Serum Ca is high (higher osteoclastic) Serum Alk-P is high

Multiple Myeloma

Serum Ca is incresaed alk-P is normal

Normal Child

Serum Ca is normal Alk-P is elevated

Paget's bone

Serum Ca is normal to high Serum Alk-P is high

SHEEHAN mnemonic?

She Hemorrhagic gland Enlarged pituitary Estrogen def. Hypotension Amenorrhea/ADH(low)/ACTH(low) Non-lactating women

After delivering a normal full-term baby, a 29-year-old woman is presented with postpartum bleeding, extreme weakness and inability to lactate. Lab evaluation confirms decreased level of FSH, LH, TSH, ACTH, and subnormal levels of T3, T4 and cortisol. This woman is most likely suffering from which of the following conditions

Sheehan syndrome

Hypocalcemia

Signs of Hypocalcemia - Muscle excitability (tetany) = - Paresthesias - Laryngospasm - Seizures - Chvostek's and Trousseau's signs

Growth Hormone Deficiency Treatment

Somatrem and somatropin Both are human GH produced by recombinant DNA technology Ineffective after closure of the epiphysis

What is the medication of choice for Conn's Syndrome

Spironolactone - it's a synthetic aldosterone receptor antagonist

Steroid-induced Myopathy

Steroid-induced myopathy is common with dexamethasone and triamcinolone Biopsy of the muscle of patients who have received long-term therapy may indicate muscle fiber atrophy. The exact mechanism of the myopathy is unknown but it is postulated that it may be due to increased protein degradation, decreased protein synthesis, and hypokalemia (and steroid-induced diabetes). The major findings of myopathy are increased serum CPK and LDH and increased urinary creatine levels

The management of myasthenia gravis requires corticosteroid therapy. However, at times the patients are presented with worsening symptoms. Why?

Steroid-induced myopathy is the problem associated with myasthenia treatment. Despite this finding corticosteroids are the mainline medication. Note: MG is an autoimmune disease and often is associated with thymoma

Steroid-induced Osteoporosis

Steroid-induced osteoporosis the 3rd most common cause of osteoporosis after menopause and aging Long-term steroid treatment causes loss of bone mass Formation of bone requires coordinated effects of the osteoclasts that break down the old bone, and osteoblasts, which lay down new bone. Effective breakdown of the old bone is a prerequisite for the formation of the new bone. It is postulated that steroids inhibit the osteoclastic function and this would distort this delicate balance and disrupt bone replacement

Arachnidonic acid => ? (cylcooxygenase)

Stopped by NSAIDs but that goes to prostacyclin (PGI2, decreases platelet aggregation, decreases vascular uterine and bronchial tone) Prostaglandin (decreased vascular and bronchial tone, Increase uterine tone and bronchial tone) Thromboxane (Increases platelet aggregation and increase vascular and bronchial tone)

Conditions increasing urinary 17-OHCS

Stress, obesity and pregnancy Ectopic ACTH and Corticosteroid therapy

Positive CRH releasing factors?

Stress; severe hypoglycemia; anxiety/fear; increase ACh and serotonin; pain; circadian rhythms

Treatment of Excess GH

Surgery Bromocriptine (dopamine agonist) Dopamine agonists - Inhibit GH release in patients with high GH - No effect in normal person

SIADH mnemonic

Syndrome of /Small cell carcinoma Increased/Intracranial Hemorrhage ADH Demeclocycline Hyponatremia/Hypouricemia

SIADH

Syndrome of Inappropriate (increased) Antidiuretic Hormone. Persistent high ADH levels, abnormal water reabsorption, expansion of ECF, hyponatremia, hypouricemia and inability to dilute urine. Causes of SIADH - Increased hypothalamic secretions - CNS disease (e.g. intracranial hemorrhage, meningitis and cerebral infarcts). - Postoperative - Ectopic non-endocrine neoplastic secretions (e.g. oat cell carcinoma) - Non-neoplastic pulmonary disease (TB, and pneumonia) Treatment - Demeclocycline (interferes with ADH effect on renal tubules) - Fluid restriction and salt intake restriction

T3 Resin Uptake (T3RU) and TBC

T3 resin uptake indirectly measures TB Capacity. It is used in calculating free thyroid hormone concentration. The T3 resin uptake test measures the number of unsaturated binding sites on the thyroid hormone transport proteins, mainly the TBG. A normalized concentration of the labeled T3 plus resin will be given to the patient The labeled T3 will bind to available sites on the serum TBG; any excess will bind to the resin. The amount that would be recovered by the resin will provide an estimate of the availability of free TBG binding sites T7 is the sum of T3RU and total T4. It reflects total free thyroxine available to tissues. T3RU has a reciprocal relationship to TBG. T7 = T3RU + T4; if T3RU increases, T4 decreases to keep T7 constant Conditions that affect Thyroid-Binding Globulin will also affect T3RU but not T7. The only two conditions that affect total thyroxine (T7) are hypo and hyperthyroidism--wherein T7 is deviated down or up accordingly

Thyroxine

T4 (thyroxine) binds to plasma binding proteins—albumin and thyroxin binding globulin (transport form that binds 75% of thyroxine) Normally over 99% of thyroxine is bound to plasma proteins; hence, free and active form is about 1% T4 is the principal thyroid hormone but T3 is the most biologically active form T4 is converted to T3 by peripheral deiodinases near or on the target tissues T3 binds to target cell nuclear receptors

Sx of fetal hyperthyroidism may be evident as early as 24th week of gestation. What are the top signs of fetal thyrotoxicosis

Tachycardia Excess fetal activity Fetal Death

Describe the classic laboratory pattern of hyperthyroidism.

The TSH level is low (unless the patient has a TSH-secreting tumor), whereas triiodothyronine (T3) and thyroxine (T4) are increased.

What causes Cushing's syndrome?

The most common cause is iatrogenic because steroids are prescribed for many different disorders. The second most common cause is Cushing's disease (a pituitary adenoma that secretes adrenocorticotropic hormone [ACTH]), which causes roughly 60% of noniatrogenic cases. Women of reproductive age are affected 5 times more often than men. Other causes include ectopic ACTH production (classically by small cell lung cancer, which is more common in men) and adrenal adenomas or carcinomas (more common in children).

21-Hydroxylase Deficiency

The most common deficiency. Congenital adrenal hyperplasia Deficiency of both mineralocorticoids and glucocorticoids. Due to lack of feedback over pituitary there is a high level of ACTH in the plasma. Pathways are shunted to sex steroid production There is a high level of androgens Adrenal virilism. Male phenotype exaggerated. Precocious puberty (<9 in boys, <8 in girls) Female patients present with masculine characteristics such as enlarged clitoris Salt loss and hypotension Mostly in men

In which condition of the 6 pregnant women (primary, secondary, tertiary hypo/hyperthyroidism) will you most likely see fetal hyperthyroidism?

The primary and secondary hypothyroid and the tertiary hyperthyroidism women because all of them have high TRH

What are the signs and symptoms of hypoparathyroidism?

The same as those for hypocalcemia (tetany, prolonged QT interval on electrocardiogram [ECG]). Calcium is low, phosphorus is high, and PTH is low.

What is thyroglobulin and is it the same compound as thyroid binding globulin or not

Thyroglobulin, an iodide-containing protein, is the stored form of thyroid hormone that entirely occurs within the colloids of the follicles of the thyroid gland. Thyroxine-binding globulin is a protein responsible for carrying the thyroxine in the blood. Note: Thyroglobulin used to be called "thyroid colloid" or "colloid

THYROID ENDOCRINOLOGY

Thyroid Stimulating Hormone (TSH) of the anterior pituitary gland stimulates follicular thyroid cells. Follicular cells produce thyroxine

HYPOTHYROIDISM - Causes of hypothyroidism

Thyroid failure (primary), pituitary failure (secondary), or hypothalamic disease (tertiary hypothyroidism) Insufficient iodine intake Hypothalamic lesion (lowered TRH/tertiary) Anterior pituitary lesion (lowered TSH/secondary) Thyroid surgery and 131I therapy Peripheral resistance to Thyroxine (autosomal recessive presented with high thyroxine and at times high TSH and goiter) Cretinism: Congenital hypothyroidism Hashimoto's Thyroiditis: Chronic lymphocytic inflammation due to autoimmune destruction of Thyroid. Often co-exists with other autoimmune diseases (e.g. diabetes, SLE)

Which hormones are most important for growth?

Thyroid hormone is the most important hormone during prenatal life and within the first 5 years. GH is most prominent hormone after 5 years of age. During sexual maturity years GH synergizes with sex hormones and causes growth spurt

Treatment of Hypothyroidism

Thyroid hormone replacement Administration of levothyroxine sodium or liothyronine sodium. In infants with hypothyroidism levothyroxine must be initiated as early as the first week of life to avoid mental retardation. Side effects of Levothyroxine: Tremor and heat intolerance

Hashimoto Thyroiditis

Thyroiditis (inflammation of thyroid) has many causes. The most common cause is Hashimoto's. Hashimoto's is also the most common cause of hypothyroidism after age 6 in the USA. Hashimoto's is associated with autoimmune diseases such as SLE, pernicious anemia, Sjogren's, diabetes and rheumatoid arthritis. Abnormal antibodies and white cells attach to the thyroid and result in an autoimmune destruction. Patients are mainly women, young, middle-aged or elderly. High levels of antibodies in the blood confirm diagnosis. Patients often have antibodies to various thyroid antigens. The most common antibodies are antithyroid peroxidase (anti-TPO), anti-thyroglobulin, (anti-Tg), and TSH receptor-blocking antibodies. Although a small percentage of patients may have no antibodies

Thyroxine functions

Thyroxine (T3) is overall calorigenic and synergizes with GH Increases Na+/K+ATPase synthesis, which causes the following changes: - Increases mitochondrial oxygen use - Increases intracellular heat formation - Increases basal metabolic rate (BMR) in all tissues except brain, spleen and gonads. Increases number and affinity of beta-adrenergic receptors in the heart and cardiac output. Increases myelination and synaptic formation in the nervous system (required before birth and during pregnancy). Increases GI absorption of glucose, increases glycogenolysis, gluconeogenesis, lipolysis and fuel availability. Increases protein degradation more than synthesis (catabolic). Increases growth hormone and somatomedin synthesis (synergizes with GH to promote bone formation)

What non-steroid hormone has intra-nuclear transduction

Thyroxine! Two different mechanisms for thyroxine: Binding to regulatory gene regions Acting through Protein kinase C

Tx of Hyperprolactinemia

- Dopamine agonists •Typically 1st line therapy •Effective in > 90% of patients •Cabergoline, bromocriptine, pergolide - Surgery •When dopamine agonist therapy has been unsuccessful or for •Giant macroadenomas (> 3cm) in woman who wish to get pregnant - Radiation therapy •Not for primary therapy, may increase risk of brain tumors •Only following postoperative debulking to prevent regrowth

What causes hypocalcemia?

- Hypoparathyroidism (usually after thyroid gland surgery) - Pseudohypoparathyroidism (genetic end-organ unresponsiveness to PTH with normal PTH levels, shortened metacarpal bones, short stature, and mental retardation) - DiGeorge syndrome - Vitamin D deficiency (osteomalacia, rickets) - Renal failure of any cause and certain renal tubular problems - Acute pancreatitis (one of the Ranson criteria) - Secondary to hypomagnesemia Hypoproteinemia of any cause may lead to low levels of total serum calcium, but levels of ionized calcium (the active form) are normal. In any patient with low serum calcium, the first step is to determine whether the serum albumin level is decreased. If it is, no treatment is required and no symptoms will develop.

What is the proposed mechanism of tetany in hypocalcemia?

- Normal Ca is bound to voltage-gated sodium channel adn slows sodium flow - In hypocalcemia lack of calcium will remove the impediment of sodium flow - This leads to production of too many muscle spasms.

acromegaly pathophys and diagnosis?

- Pathophysiology •GH-secreting tumor (almost always pituitary) •Excess IGF-1 secretion -> most clinical manifestations - Diagnosis •Elevated GH and IGF-1 levels •Glucose load will NOT suppress GH secretion •MRI of the head showing pituitary adenoma

Adverse effects of calcium salts

- Peripheral vasodilation - Cardiac arrhythmias (with rapid infusion)

What is the role of 11-OH-hydroxysteroid dehydrogenase enzyme and why patients who are deficient in this enzyme have metabolic alkalosis?

11 B-HSD catalyses conversion of inactive 11 keto-product cortisone to active cortisol and regulates the access of glucocorticoids to steroid receptors Two types exist; type 1 and 2 The mineralocorticoid receptors in the collecting duct are acted upon by both aldosterone and cortisol Cortisol has a much higher affinity for these receptors. Normally 11 B-HSD converts cortisol to cortisone in the collecting ducts and inhibits the effect of cortisol. This will allow the less potent aldosterone to become the main modulator of the receptors. In the recessive condition of 11 B-HSD2 deficiency, the cortisol with higher affinity for mineralocorticoid receptors build up and causes excessive sodium retention and H+ excretion, HTN, and hypokalemia alkalosis These patients characteristically have low serum aldosterone and renin levels.

ADRENOCORTICAL DISEASES A 17-year-old girl is being admitted to the OB/GYN clinic for the complaint of amenorrhea. Examination reveals lack of development of secondary sex characteristics. Blood pressure is 180/110; result of funduscopy is normal; she has no lymphadenopathy or hepatosplenomegaly. Blood analysis indicates hypokalemia, hypernatremia, metabolic alkalosis, and suppressed renin. Urinary analysis confirms a high level of urinary gonadotropins What's the most likely cause of this?

17-OHase deficiency So you don't make sex hormones and you don't make cortisol because you cannot get to 17-OH progesterone. High level of urinary gonadotropins (FSH, LH) because there is no estrogens to suppress it.

Grave's Disease - info

AKA. Diffuse Toxic Goiter The most common cause of hyperthyroidism Autoimmune Response - A Type II hypersensitivity disease with the presence of autoantibodies called Long-Acting Thyroid Stimulators (LATS). Two important LATS are TSI and TGI. - Mechanism: T-cells are sensitized to thyroid antigens. They stimulate B-cells to form thyroid specific antibodies such as "Thyroid-stimulating immunoglobulin (TSI)". - TSI (an IgG) is a potent TSH receptor agonist and stimulates thyroxine formation. - The other immunoglobulin that causes follicular hyperplasia is Thyroid Growth Immunoglobulin (TGI)

Kallmann's Syndrome

AKA. Hypogonadotropic hypogonadism Congenital or acquired gonadotropin deficiency. May be acquired by pituitary infection, infarction, trauma or tumors. Lack of FSH and LH. LH-releasing hormone defect Lack of secondary developments. Amenorrhea, and lack of breast development in females. Cryptorchidism in males. Indefinite stay in pre-puberty. Mental retardation. Patients are presented with ANOSMIA (hypoplasia of olfactory lobe), CLEFT PALATE (midline development defect) and COLOR-BLINDNESS Treatment for secondary traits: Sex hormones; presents in males and females

Somatomedin

AKA. Insulin-like Growth Factor-1 (IGF-1) Production Site: Mainly liver Positive Releasing Factors: Increased growth hormone Actions of Somatomedin - Most Tissues: Insulin-like activity - Muscle: Increases AA uptake and protein synthesis; increases lean body mass - Bone: Increases osteoblastic activity; Increases thickness of epiphyseal end-plate; and increases linear bone growth - Inhibits both glucagon and insulin secretion from pancreas

HYPERTHYROIDISM

AKA. Thyrotoxicosis Increased metabolic rate and weight loss Increased heat production, warm skin, sweating and heat intolerance. Oligomenorrhea and amenorrhea Weight loss despite increased appetite. Negative nitrogen balance (muscle wasting) Separation of fingernails from finger bed (Plummer's nail) Hand tremor and starred eyes. Tachycardia,dyspnea, increased cardiac output, wide pulse pressure and cardiomegaly. Disorders associated with thyrotoxicosis: Grave's disease; Excess TSH (rare); Thyroid adenomas; Toxic goiter Laboratory Findings: High T3 and T4; low TSH Increased radioactive iodine uptake by the thyroid gland

What is a hot nodule?

About 5% of people with hyperthyroidism have only a single tumor that makes too much thyroxine. Abundance of thyroxine will inhibit TSH and as a result the rest of the thyroid gland may shut down while the nodule keeps on pouring thyroxine in the blood. Note: Nodules can be hot or cold; thyroid gland absorb radioiodine and the outline of the gland appears butterfly like. If the nodule is due to non-thyroxine making cells they appear lighter (cold)

TUMORS OF ADRENAL MEDULLA GANGLIONEUROMA

All benign Mostly children < 5 Slight increase in catecholamines Abd mass

Potency of Various Steroid

All steroids have both mineralo and glucocorticoid effects. Cortisol and cortisone have equal mineralo and glucocorticoid effects. Dexamethasone by far is the most potent glucocorticosteroid, and fludrocortisone is the most selective mineralocorticoid

Structure of classic NSAIDs?

Allow them to fit into both COX active sites and cause inhibition. COX-2 specific inhibitors (Vioxx) and Celebrex have side chains that fit into COX-2 active sites only, thereby inhibiting production of inflammatory mediators without compromising normal functions of kidney, GI, and platelets

What are the three notable drugs that inhibit adrenal cortex?

Aminoglutethimide - aromatase inhibitor - Inhibits P-450. In turn inhibits desmolase activity - It is used in (1) Cushing's treatment and (2) to reduce estrogen level in breast cancer Ketoconazole: Desmolase inhibitor and antifungal. Reduces all steroids. Inhibits P450 Spironolactone: Potassium sparing Diuretic used for treatment of hypertension and primary hyperaldosteronism (when zona glomerulosa is overactive)

Factors associated with increased T3RU and decrease TBG?

Androgens Adrenal Excess Albuminuria ASA Acute illness Ailing liver

Factors Associated With Decreased TBG And Increased T3RU

Androgens decrease TBG Glucocorticoids (Cushing's/Adrenal Excess) Increased cortisol suppresses TSH and causes low total T4. Hence, TBC decreases and T3RU increases. Albuminuria: Nephrosis (nephrotic syndrome). Severe cases are associated with urinary TBG loss. Aspirin: Competes with T4 & T3 for TBG Active Acromegaly Acute Illnesses Ailing Liver (chronic liver disease). TBG synthesis affected.

Primary Sources of Ketosteroids

Androstenedione Androsterone Testosterone and Dihydrotestosterone Dehydroepiandrosterone (DHEA)

17-OH progesterone => ?

Androstenedione => Testosterone

EFFECTS OF CALCITONIN ON BONE

Antagonizes PTH action and directly inhibits bone resorption. Decrease osteoclastic activity and decrease resorption. Increases osteoblastic activity and increases mass deposition

Dermography - Grave's Disease

Any age, peaks at 20-40 year-old Familial disposition. Associated with HLA-DR3 and HLA-B8. More common in women (10:1 female to male ratio). More common in Caucasians, Japanese and Chinese

What is apoplexy?

Apoplexy is uncontrolled bleeding into the brain; usually as a result of cerebrovascular accidents.

Myxedematous Changes in the Heart

Are due to accumulation of GAGs and they include: cardiomegaly, decreased contractility, pericardial effusion, and decreased pulse rate and cardiac output

What is the arterial blood gas finding in myxedema coma?

Arterial blood gases often reveal respiratory acidosis, hypoxia and hypercapnia

Almost everyone knows me. If you get too much of me, I will give you respiratory alkalosis and metabolic acidosis. I am expert at wiping out prostaglandins. If you are in pain or have an inflamed musculoskeletal tissue, quite often you come to me for help. I don't get along quite well with kids; especially those who have chicken pox or influenza B. If I get into their body I will ruin their liver and swell their heads!

Aspirin

Aspirin inhibits both COX 1 and 2. What would be a reasonable explanation for the finding that higher dosages of aspirin are needed for anti-inflammatory than analgesic effects?

Aspirin at low doses inhibits COX 1 and at higher doses COX-2

What is the difference between aspirin and salicylic acid?

Aspirin is acetylsalicylic acid

MOA of aspirin?

Aspirin is rapidly acted upon by esterases in the body and produces salicylic acid which has antipyretic, anti-inflammatory and analgesic properties. The acetate moiety of aspirin prolongs these functions by inactivating cyclooxygenase permanently

17-Hydroxylase Deficiency

Autosomal recessive defect Congenital adrenal hyperplasia Conversion of progesterone to subsequent sex hormones and glucocorticoids is affected. Female characteristics are preserved, but secondary developments are affected. Pathways are shunted to further mineralocorticoids production. Increased mineralocorticoids leads to sodium and water retention and hypertension Boys due to lack of androgens will have ambiguous genitalia with increased gonadotropins and low testosterone. External genitalia will look ambiguous. hermaphrodite features. Mostly in women

11-Hydroxylase Deficiency

Autosomal recessive deficiency that causes deficiency of corticosterone, aldosterone and cortisol. Increased levels of deoxycorticosterone leads to salt and fluid retention, and hypertension. Hypertension is due to accumulation of deoxycorticosterone which has a very strong mineralocorticoids effect. Like 21-alpha-hydroxylase leads to adrenal virilism (AKA. adrenogenital syndrome) and masculinization. Less common than 21-hydroxylase deficiency Note: Likewise 21-hydroxylase deficiency male phenotypic characteristics are promoted. Female patients have masculine characteristics

Treatment for Thyroid Storm

Beta-blockers PTU Methimazole Glucocorticoids (inhibit peripheral T4 to T3 conversion)

Long Acting Glucocorticoids

Betamethasone, dexamethasone, paramethasone 1 -3 days

CORTISOL

Binds to plasma proteins—Transcortin and albumin Bound hormone is inactive. Transcortin binds 75% Estrogen and thyroxine increase transcortin but insert no change on free cortisol concentration Androgens decrease transcortin Plasma levels are high immediately prior to waking and lowest around midnight Produced in Zona Fasciculata and Reticularis Release is stimulated by ACTH and epinephrine Signal transduction is via nuclear receptors (gene transcription). Cortisol is metabolized by mixed function oxidases (MFO's) in the liver

OSTEOPOROSIS

Bone resorption rate is more than deposition Reduced bone density Vertebrae, wrist, hip, humerus and tibia are prone to fractures Associated With: - Decreased plasma estrogen - Decreased weight bearing physical activity - Decreased plasma Vitamin D - Increased lean body mass - Alcoholism, and smoking Demography - Net bone mass loss after age 50. - White female > Black female > White male > Black man Diagnosis - Ballooning of intervertebral disks on X-ray, compression fracture Treatment - Stabilize bone mass by reducing rate of loss. - Estrogen-replacement therapy. - Weight-bearing physical activity.

Acetaminophen and aspirin share all of the following EXCEPT

Both are anti-inflammatory

Patient with renal failure are presented with metabolic acidosis. What type of acidosis anion or non-anion gap would you expect in these patients?

Both! Bicarbonate reabsorption defect causes non-anion gap acidosis but loss of ability to excrete ammonium and organic acid causes anion gap

What stimulates Phopholipase A2?

Bradykinin and angiotensin

How is thyroid eliminated?

By cytochrome P450 via liver

Hypoparathyroidism

Causes - Most common cause is accidental removal during surgical neck exploration or thyroidectomy - DiGeorge Syndrome: Congenital absence of the parathyroid gland Characteristics - Hypocalcemia; causes tetany in distal musculature (e.g. Trousseau's sign) - Hyperphosphatemia - Decreased renal phosphate excretion Diagnosis: Hypocalcemia plus hyperphosphatemia in the presence of normal renal function Treatment: Administration of PTH, vitamin D, and Calcium

CUSHING'S SYNDROME

Causes: - Exogenous: iatrogenic due to chronic glucocorticosteroid excess - Endogenous Causes: bilateral adrenocortical hyperplasia or tumors Demography: More in females (ratio 8:1). Age of onset is 20-40 years. Hormonal Characteristics: - Excess glucocorticoids, androgen and mineralocorticoid-like effects - Increased plasma and urine free Cortisol Clinical Correlations - Increased cortisol causes hyperglycemia - Mineralocorticoid-like effects cause increased Na+retention, volume expansion and hypertension. - Hypokalemia, Hypochloremia, and metabolic alkalosis (All 3 more common with ectopic ACTH) - Excess cortisol causes bone resorption and osteoporosis Other Characteristics - Truncal obesity - Hair growth - Poor wound healing - "Buffalo hump" and "Moon facies Cushing's disease: so called if hyperplasia of adrenals is due to hypersecretion of pituitary ACTH

Conn's syndrome

Causes: Unilateral adenoma or hyperplasia of zona glomerulosa Demography: More in females (male to female ratio 2:1). Age of onset, 30-50 years Hormonal Characteristics: Excess aldosterone CLINICAL CORRELATIONS - Increased aldosterone causes increased Na+retention, increased K+excretion, and increased acid secretion by the kidneys. - ECF volume expansion and hypertension. - Hypokalemia and metabolic alkalosis Hallmark: Decreased renin level due to hypertension

Hyperparathyroidism Cause?

Chief cell adenoma or hyperplasia of parathyroid gland lead to hyper-secretion of PTH

What's the pathophysiology of osteitis fibrosa cystica

Chronic kidney failure => alpha-1 hydroxylase deficiency => Vit D deficiency => Hypocalcemia => High PTH => Secondary hyperparathyroidism, Chronic bone resorption, Osteoporosis, cystic demineralization of bone.

Thyroid Binding Globulin (TBG)

Concentration in plasma varies under different conditions. Factors Associated With Increased TBG - Pregnancy, and oral contraceptives (estrogens) - Active infectious hepatitis and biliary cirrhosis - Drugs: Tamoxifen and perphenazine - Acute Intermittent Porphyria Factors Associated With Decreased TBG - Androgens - Glucocorticoids - Nephrosis - Chronic liver disease - Active acromegaly

Hydroxysteroid Dehydrogenase Deficiency

Congenital adrenal hyperplasia Lack of steroids removes negative feedback over the pituitary and ACTH production stays uninhibited. Constant ACTH production causes hypertrophy of the adrenal cortex. Pregnenolone is not converted to all other subsequent steroids. All pathways; mineralocorticoids, corticosteroids and glucocorticoids are affected The second most fatal steroid biosynthetic deficiency. Early death

Cretinism

Congenital/neonatal hypothyroidism Causes: - Maternal iodine deficiency - Maternal hypothyroidism (low thyroxine) - Developmental and genetic defects of fetus - Transplacental transfer of antithyroid antibodies from mother with autoimmune hypothyroidism. Thyroxine is required for CNS and physical growth. Fetus is most sensitive to (and most dependent on) maternal T3 and T4 deficiencies during the first and second trimesters.

Type 1 will do what?

Converts inactive form of cortisone to cortisol (active) => mineralocorticoid receptors and causes hypernatremia, hypokalemia, HTN, alkalosis

Significance of 17-hydroxycorticosteroid

Cortisol is metabolized in the liver to inactive products of 17-hydroxycorticosteroid (17-OH-CS) and excreted in urine. High levels of 17-OHCS are indicative of hypercortisolism

What is an easy way to remember cortisol?

Cortisol is the stress hormone. Its blood level rises just before waking up, and just before getting ready to confront everyday's challenges!

Why dexamethasone has to be done at a specific time of the day?

Cortisol level is affected by circadian rhythm

Bartter's diagram

Decrease Renal Aldosterone sensitivity => Decreased Na+ reabsorption (hypotension) => Renal Na loss => JG hyperplasia and increase renin => Increased aldosterone => Hypokalemia and alkalosis

PGI2? (vessel wall)

Decrease platelet aggregation and vasodilates

PGs

Decrease vascular and bronchial tone Increase uterine tone and bronchial tone

Dx of Hypothyroidism

Decreased T4 (and T3) is the common denominator of all hypothyroidisms Increased plasma TSH happens in primary hypothyroidism. Lack of Thyroxine removes negative feedback on hypothalamus and anterior pituitary. TSH is decreased or normal in secondary hypothyroidism. Decreased T3 resin uptake Other Findings: Low amplitude QRS, and flattened or inverted T wave on the ECG

Causes of GH Insufficiency

Decreased hypothalamic GHRH Decreased output of GH Reduced synthesis of somatomedin GH receptor dysfunction Characteristics: Short stature and delayed puberty

Characteristics of Hypothyroidism

Decreased metabolic rate and weight gain. Decreased heat production, cool skin and cold sensitivity. Dry hair, dry skin and periorbital puffiness Stiffness, cramping of muscles, and carpal tunnel syndrome Bradycardia and decreased cardiac output Hypoventilation and hypercapnia Decreased GFR Slow physical, mental and intellectual activities Decreased appetite and increased weight Decreased intestinal peristalsis (constipation) Positive nitrogen balance

Cortisol and the Adipose Tissues

Decreases glucose uptake, increases lipolysis, increases free fatty acid (FFA) export (FFAs go to the liver and they are converted to acetylcholine, the precursor of ketone bodies) and increases plasma glycerol level. Catabolic role

CORTISOL EFFECTS ON THE MUSCLE

Decreases glucose uptake, increases proteolysis and increases AA export (amino acids go to liver), decreases protein synthesis. Glucagon-like effect Catabolic role on the muscles

Which of the following hormones is expected to increase in this patient's plasma

Deoxycorticosterone and corticosterone

Progesterone => 21-hydroxylase => ? => 11-hydroxylase =>? => aldosterone synthase ?

Deoxycortisone => Corticosterone => Aldosterone

What is the desirable total cholesterol to HDL ratio?

Desirable ratio of cholesterol to HDL is less than 3.5

What is the desirable serum concentration of HDL?

Desirable serum concentration of HDL is more than 35-40 mg/dL

What is the desirable serum concentration of LDL?

Desirable serum concentration of LDL is less than 130 mg/d

What is the desirable level of triglycerides?

Desirable serum triglyceride level is less than 150

What's the most fatal steroid deficiency? Many patients die in early infants...

Desmolase deficiency

Sheehan's syndrome Dx and tx

Diagnosis •Can present similarly to pituitary adenoma, apoplexy, or hypophysitis •Low prolactin and lack of response to TRH helps differentiate sheehan's from apoplexy and hypophysitis Treatment •Hormone replacement •High dose steroids (hypophysitis, apoplexy) •Decompressive surgery (hypophysitis, apoplexy)

How does one confirm the diagnosis of insulinoma?

Diagnosis of insulinoma is suggested by hypoglycemia and elevated insulin, pro-insulin and C-peptide levels and confirmed by medical imaging

What are the three medications used to treat insulinoma?

Diazoxide, somatostatin and streptozotocin (a nitrosourea)

Diflunisal, a derivative of salicylic acid, is not metabolized to salicylate. It is much more potent than aspirin in analgesic and anti-inflammatory effects, but it has no antipyretic effects. Why?

Diflunisal cannot enter the CNS

ACTH PRODUCING TUMORS TOPIC

Do not have truncal obesity because malignancies cause weight loss Hypokalemia, hypochloremia, and metabolic alkalosis Thymomas Oat cell carcinoma Pheochromocytoma Islet cell tumor of pancreas Carcinoid tumors

Three common side-effects of salicylates, acetaminophen, phenylpropionic acids are GI bleeding and hepatorenal toxicity. Why they cause renal toxicity?

Due to reduction of PG in kidney Reduced PG = reduced renal flow and function Resultant salt and water retention may reduce effectiveness of medications.

Summary of Side Effects of NSAIDs

Epigastric distress (related to the inhibition of PG and damage to the mucoid coating) Coagulation disorders Uncoupling of oxidative phosphorylation Hypersensitivity and urticaria (especially with aspirin) Reye's in children with viral infection

ADRENAL MEDULLA

Epinephrine, derived from tyrosine is made by the chromaffin cells of the adrenal medulla. Stimulatory Release Factors - Increased SNS drive - Increased stress and pain - Increased cortisol - Hypoglycemia and hypovolemia Epinephrine actions in most tissues depend on the type, location and distribution of receptors. Overall, it mobilizes fuels and increases plasma glucose, FFA and ketoacids. Effects on the Liver - Increases lipolysis (decreases lipogenesis) - Increases gluconeogenesis (decreases glucose uptake). - Increases glucose export and increases glycogenolysis Effects on the Adipose Tissue - Increases lipolysis - Blocks insulin action Effects on Skeletal Muscle - Blocks insulin action - Decrease glucose uptake - Increase proteolysis - Increase amino acid export Signal Transduction for Epinephrine - Linked to G protein - alpha-1: C kinase - alpha-2: GiA kinase - beta-1: GsA kinase; - beta-2: GsA kinase

What medication with a prefix similar to etidronate, drops blood calcium level?

Ethacrynate A loop diuretic

What other similar-sounding mediation to etidronate is helpful in reducing osteoarthritic pains?

Etodolac An indoleacetic acid, NSAID, similar to indomethacin

Growth Hormone Abnormalities

Excess Growth Hormone. Often due to somatotropic cell adenomas. Usually due to tumor of somatotrophs in the anterior pituitary that cause increased GH secretion. Prior to Puberty and Closure of Epiphyses: Gigantism (excessive linear growth) After Puberty: Acromegaly Treatment: Somatostatin

What is the reason for hirsutism in female Cushing's patients?

Excess androgenic effects

Iatrogenic causes (cortisol therapy)

Exogenous cortisol is up - primary cause ACTH is down No effect of dexamethasone effect

PGE

Expands (dilates) bronchioles Slows stomach acid Prolongs PDA closure Increases Uterine tone

What are the two tendons affected in DeQuervain's syndrome?

Extensor pollicis brevis and abductor pollicis longus Extensor pollicis brevis fx and abductor pollicis longus - radial abduction thumb

22 yo caucasian woman is presented with a 2 year hx of constipation and fatigue. Serum analysis is significant for hypercalcemia, high parathyroid hormone, and hypermagnesemia. On further eval, she denies history of kidney stones, however she mentions that her mother has been dx with hypercalcemia at age 45. Which of the following is dx?

Familial hypocalciuric hypercalcemia A rare and for most part asx condition Important in differential diagnosis of primary hyperparathyroidism. Is more benign compared to primary hyperparathyroidism and in contrast to the latter is neither responsive to parathyroidectomy nor requires parathyroidectomy. AD defect of CASR gene that encodes for calcium-sensing receptor; a G-protein receptor expressed mainly in kidneys and parathyroid glands Loss of CASR mediated feedback inhibition of PTH in response to hypercalcemia, and feedback inhibition of calcium reabsorption in nephrons in reponse to hypercalcemia tetrad of findings: Hypercalcemia, Hypocalciuria, high PTH, and Hypermagnesemia Dx: PCR DNA testing

Surgery for parathyroid adenoma if these are met...

1. Serum calcium > 1 mg/dL above normal 2. Hypercalciuria > 400 mg/day 3. Reduced creatinine clearance by >30% 4. Reduced bone density below T score of -2.5 at site 5. Age <50 years

What are the anterior pituitary hormones?

Anterior Pituitary hormones are FSH, LH, TSH, GH, ACTH, and Prolactin

What is the C-peptide level of a type 2 diabetic; low or high or normal?

C-peptide levels in type 2 diabetics are variable and can be normal, reduced or even increased.

What causes central DI?

Central DI is caused by a lack of ADH production by the posterior pituitary. Although it is often idiopathic, look for trauma, neoplasm, or sarcoid/granulomatous disease as the cause. Order a CT or MRI scan of the head, if indicated

What is Cushing's disease?

Cushing's disease is hypercortisolism resulting from an ACTH-hypersecreting pituitary adenoma.

What is lusitropy?

Lusitropy describes myocardial relaxation in the light of increased myocardial contractility, as a result of sarcoplasmic calcium release.

What are the posterior pituitary hormones?

Posterior Pituitary hormones are: Oxytocin and ADH

Treatment of Grave's

Surgery •Pros: Successful cure •Cons - Hypothyroidism - Hypoparathyroidism => hypocalcemia - Laryngeal nerve damage •Radioiodine ablation •Thioamides - Propylthiouracil (PTU) and methimazole (MMI) - Actively transported into thyroid - Inhibit iodine organification and iodotyrosine coupling - Approximately 30% spontaneous remission within first 2 years - MMI usually preferred due to less side effects (except during pregnancy)

What are the symptoms and signs of hypoadrenalism (Addison's disease)?

Symptoms: Anorexia, weight loss, weakness, apathy. Signs: Hypotension, hyperkalemia, hyponatremia, hyperpigmentation (only if the pituitary is functioning because of melanocyte stimulating hormone), nausea and vomiting, diarrhea, abdominal pain, mild fever, hypoglycemia, acidosis, eosinophilia, and shock

What are the most common causes of hyperthyroidism?

The most common cause is Graves disease, which is characterized by a diffusely enlarged thyroid gland, positive thyroid-stimulating immunoglobulins and antibodies, exophthalmos, proptosis, ophthalmoplegia, and pretibial myxedema. In older patients, look for toxic multinodular goiter (individual lumps instead of diffuse enlargement of the gland and "hot" nodules on thyroid nuclear scan). Other causes include adenoma (single lump that is "hot" on nuclear scan), subacute thyroiditis (viral infection with tender, painful thyroid gland), and factitious hyperthyroidism (in which the patient takes thyroid hormone). Rare, exotic causes include amiodarone (which can cause hypothyroidism or hyperthyroidism), TSH-producing pituitary tumor, thyroid carcinoma, and struma ovarii (an ovarian teratoma that secretes thyroid hormone).

What is Whipple's triad?

Whipple's triad encompasses the three conditions which must be met to confirm a clinical suspicion of hypoglycemia as the underlying cause of a person's symptoms. These are: (1) Low serum glucose; (2) symptoms related to low blood glucose; and (3) resolution of symptoms after normalizing blood glucose

Sheehan's syndrome Presentation

•Failure of lactation •Amenorrhea •Adrenal insufficiency •Hypothyroidism •Diabetes insipidus in approximately 5% of all cases.

Physiological Actions of Thyroid Hormone

•Increased basal metabolic rate (BMR) •Increased expression and activity of Na+, K+/ATPase pump •Potentiation of catecholamine actions •Upregulate expression of β-adrenergic receptors •Direct effects on the heart Differences between T4 and T3 •T4 is much less potent than T3 but has longer plasma half-life •T4 much more abundant than plasma T3 •T4 principally responsible for feedback inhibition of hypothalamus and pituitary

Overview of Prolactinoma

•Most common hypersecreting pituitary tumor •Microadenoma < 1 cm - 95% do not enlarge over 4-6 years of observation and may not require treatment - Therapy indicated if they enlarge or due to symptoms of hyperprolactinemia •Macroadenoma > 1 cm: Usually treated because likely to continue to enlarge and cause neurological symptoms

Sheehan's Syndrome Pathophysiology

•Pituitary necrosis following postpartum hemorrhage and hypotension •Sudden changes in arterial pressure => vasospasm => apoplexy (bleeding) •Posterior pituitary typically is spared •All anterior hormones are decreased

Summary of Synthesis of Thyroid Hormone

•Plasma iodide ion (I-) taken up by follicular cells and extruded into follicular lumen •Iodide ion (I-) oxidized to iodine (I2) by peroxidase •Iodine attached to tyrosine residues (organification step) attached to thyroglobulin forming MIT and DIT •Coupling of MIT and DIT forms T4 and T3 •Endocytosis of thyroglobulin from colloid •Cleavage of T3/T4 from thyroglobulin and release into plasma

Give the classic clinical description of a pheochromocytoma. How is it diagnosed?

Look for wild swings in blood pressure (with some measurements dangerously high), tachycardia, postural hypotension, headaches, sweating, flushing, dizziness, mental status changes, and/or a feeling of impending doom (like a panic attack). The screening test is a 24-hour urine collection for metanephrines, homovanillic acid, and/or vanillylmandelic acid (catecholamine breakdown products that are abnormally elevated in the urine). If levels are high, order an abdominal CT scan to look for an adrenal mass (Fig. 10-3). Surgical tumor removal is the treatment of choice after stabilization with alpha and then beta blockers.

What are major findings in MEN I (Wermer's syndrome)?

MEN I is associated with Parathyroid hyperplasia, Pituitary adenomas, Peptic ulcers and Pancreatic neoplasms

What are major findings in MEN IIa (Sipple's syndrome)?

MEN IIa is associated with Parathyroid adenoma, Pheochromocytoma, Medullary Thyroid carcinoma (MPH)

MEN IIb?

Medullary thyroid carcinoma Marfan's Mucosal and digestive neurofibromatosis Pheochromocytoma (MMMP)

What specific problems are caused by obesity?

Obesity causes an increase in overall mortality (at any age) and increases the risk for insulin resistance and diabetes, hypertension, hypertriglyceridemia, coronary artery disease, gallstones, sleep apnea and hypoventilation, osteoarthritis, thromboembolism, varicose veins, and cancer (especially endometrial cancer)

Acromegaly tx

Octreotide, a somatostatin analogue

MMI versus PTU

PTU •Inhibits extrathyroidal conversion of T4 to T3 •Serum half life 75 minutes •Requires bid dosing MMI •Serum half-life of 4-6 hrs •Intrathyroidal concentration ~ 100:1 remains high for approx 20 hrs •Perchlorate discharge tests which measure inhibition of iodine organification, show that MMI is better than PTU •Effective with qd dosing

Hyperprolactinemia - causes?

Physiologic Causes: Pregnancy, excessive nipple stimulation, stress •Decreased hypothalamic dopamine secretion - Hypothalamic tumors - Infiltrative (sarcoidosis) - Sectioning of pituitary stalk (head trauma, surgery) - Dopamine antagonists (antipsychotics, metoclopramide) •Prolactinoma •Other causes - Hypothyroidism - Chest wall injury - Chronic renal failure - Macroprolactinemia (decreased clearance and big prolactin) - Idiopathic

What is C-peptide and what is the clinical value of measuring its plasma level?

Plasma c-peptide blood is a measure that determines the production of and the level insulin in the subjects. Insulin is produced by the beta cells in the form of proinsulin, wherein the alpha and beta chains of active insulin are linked by a polypeptide chain, the so called "connecting peptide or c-peptide". Naturally, for every plasma molecule of insulin one expects to see one c-peptide, and level of c-peptide can be used as an indicator of beta cell insulin production. As such in those conditions wherein injected exogenous insulin may be present and mixed with endogenous insulin the c-peptide test would help to assess if high blood glucose is due to reduced insulin production or low glucose uptake by the cells.

What is the most common type of hypoadrenalism?

Secondary (iatrogenic) hypoadrenalism because of steroid treatment. People who are removed from long-term steroid therapy may be unable to secrete an appropriate amount of corticosteroids in response to stress for up to 1 year. Watch for the classic postoperative patient who crashes (with hypotension, shock, and hyperkalemia) shortly after surgery and has a history of a disease requiring steroid therapy within the past year. You may assess ACTH (usually high) and cortisol levels (inappropriately low) to help make the diagnosis, but do not wait for the results to give steroids. The patient may die. Give prophylactic stress doses of corticosteroids in the setting of an illness, operation, or other stressor to prevent an adrenal crisis.

What causes secondary hyperaldosteronism?

Secondary hyperaldosteronism is much more common than primary disease. It occurs because of low perfusion of the kidney, as in congestive heart failure, renal artery stenosis (bruit), dehydration, nephrotic syndrome, and cirrhosis. The key mechanism is that the kidney senses hypoperfusion and secretes renin; therefore, the renin level is high. Treatment of the underlying disorder (if possible) resolves the hyperaldosteronism. Potassium levels may be normal or even high. Of note, hyperkalemia may be the cause of increased aldosterone release just as hypocalcemia causes increased release of PTH. Both are normal physiologic responses.

How is hyperthyroidism treated?

Short-term (stabilizing) treatment: Propylthiouracil (PTU) and methimazole/carbimazole can be used as suppressive agents. Beta blockers are used in the setting of thyroid storm (severe hyperthyroid state—an emergency). Iodine can also suppress the thyroid gland but is rarely used for this purpose clinically. Definitive (curative) treatment: Radioactive iodine ablation of the thyroid gland is typically used. Surgery is preferred in pregnant patients. Hypothyroidism may result from either treatment; if so, it is treated with thyroid hormone replacement (for life).

What are the symptoms and signs of primary hyperaldosteronism (Conn syndrome)? What are the causes?

Symptoms: Weakness and edema. Signs: Hypertension, hypokalemia, hypernatremia, and edema. Conn syndrome is caused by an aldosterone-secreting adrenal neoplasm. Because it is a primary disease, renin levels are low; the rest of the endocrine axis responds appropriately to gland dysfunction. Order a CT scan of the abdomen to look for an adrenal mass. The treatment is surgical removal of the tumor.

What are the symptoms and signs of hypothyroidism?

Symptoms: Weakness, lethargy, fatigue, cold intolerance, weight gain with anorexia, constipation, loss of hair, hoarseness, menstrual irregularity (menorrhagia is classic), myalgias and arthralgias, memory impairment, and dementia. Always rule out hypothyroidism as a cause of dementia. Signs: Bradycardia; dry, coarse, cold, and pale skin; periorbital and peripheral edema; coarse, thin hair; thick tongue; slow speech; decreased reflexes; hypertension; carpal tunnel syndrome and paresthesias; vitiligo, pernicious anemia, and diabetes (remember the autoimmune association between these three conditions and Hashimoto disease); and coma (severe disease). In children, congenital hypothyroidism may occur (mental, motor, and growth retardation).

What are the symptoms and signs of Cushing syndrome (increased corticosteroids)?

Symptoms: Weight gain, changes in appearance, easy bruising, acne, hirsutism, emotional lability, depression, psychosis, weakness, menstrual changes, sexual dysfunction, insomnia, and memory loss. Signs: Buffalo hump, truncal and central obesity with wasting of extremities, round plethoric facies, purplish skin striae, acne, hirsutism, weakness (especially of the proximal muscles), hypertension, depression, psychosis, peripheral edema, poor wound healing, glucose intolerance or diabetes, osteoporosis, and hypokalemic metabolic alkalosis (because of mineralocorticoid effects of certain corticosteroids). Growth may be stunted in children.

What kidney disease has strong association with von Hippel-Lindau disease?

The kidney disease that is strongly associated with von Hippel-Lindau is renal cell carcinoma.

What causes hypoparathyroidism?

The most common cause is accidental removal or damage during thyroid surgery. Watch for tetany after thyroid surgery. Rare causes are genetic. Watch for DiGeorge syndrome in children with congenital absence of parathyroid glands, tetany in the first 48 hours of life, absent thymus gland, immunodeficiency, cardiac anomalies, and midline facial defects.

What are the main primary and secondary causes of Addison's disease?

The most common cause of Addison's is autoimmune adrenalitis. Other causes include tuberculosis (used to be the most common cause 10% of adult cases), Waterhouse-Friedreich's syndrome as a result of Neisseria meningitidis, bilateral adrenalectomy, hemorrhage into the adrenal gland, neoplasms, and infections (histoplasmosis and cytomegalovirus). Abrupt withdrawal of long-term corticosteroid and pituitary hypofunction may cause secondary Addison's.

What are the common causes of hypothyroidism?

The most common known cause is Hashimoto's thyroiditis. Women of reproductive age are affected 8 times more often than men. Histology reveals lymphocytes in the thyroid gland as well as antithyroid and anti microsomal antibodies. Other autoimmune diseases may coexist. The associated goiter is nontender. The second most common cause is iatrogenic after treatment of hyperthyroidism. Other less common causes include iodine deficiency, amiodarone, lithium, and secondary hypothyroidism because of pituitary or hypothalamic failure (look for decreased TSH), such as with Sheehan's syndrome (hypopituitarism caused by pituitary necrosis from blood loss and hypovolemic shock during and after childbirth).

What are the other causes of hypoadrenalism?

The most common primary (non-iatrogenic) cause is autoimmune (idiopathic) disease. Patients may have other autoimmune diseases, such as hypothyroidism, pernicious anemia, vitiligo, diabetes, or hypoparathyroidism. Other causes include metastatic cancer (especially lung cancer), infection (tuberculosis, fungal infections, and opportunistic infections in AIDS and other immunosuppressed states), ketoconazole, and pituitary/hypothalamic failure.

Define the syndrome of inappropriate antidiuretic hormone secretion (SIADH). How is it diagnosed?

The name says it all: ADH is released inappropriately. SIADH is a consideration in patients with hyponatremia and normal volume status (euvolemic). In SIADH, serum osmolarity is low, but urine osmolarity is high (inappropriate urine concentration). Look for the values of all electrolytes and lab tests to be low (the classic example is uric acid) because of dilution of the serum with free water secondary to inappropriate ADH.

What are the symptoms and signs of hyperparathyroidism?

The same as those for hypercalcemia ("bones, stones, groans, and psychiatric overtones";). In primary cases, serum calcium is high, phosphorus is normal to low, and parathyroid hormone (PTH) is increased. In secondary cases, calcium is low.

How is precocious puberty different from pseudoprecocious puberty?

True precocious puberty is usually idiopathic but can be caused by central nervous system (CNS) lesions. A general rule of thumb is that true precocious puberty causes testicular or ovarian enlargement, which does not occur with pseudoprecocious puberty (ovarian cysts are not considered true ovarian enlargement). All patients with suspected precocious puberty should have a gonadotropin-releasing hormone (GnRH) stimulation test. If a dose of GnRH produces the typical pubertal response of increased follicle-stimulating hormone and luteinizing hormone, true precocious puberty is diagnosed. An MRI scan of the brain should be obtained to rule out CNS disease (e.g., hamartomas, tumors, cysts, trauma) as the cause.

Dx of Pheo

•1st discontinue interfering medications •24 hr urine collection for free catecholamines and metanephrines, if elevated then plasma total catecholamines (norepinephrine and epinephrine) to confirm diagnosis. •Clonidine suppression test for indeterminate values •Clonidine is administered > 12 hrs after antihypertensive drugs discontinued •Plasma total catecholamines measured before and 3 hrs post clonidine administration •In patients without pheochromocytoma, catecholamine level should drop to < 500 pg/mL after clonidine administration

Pathophysiology of Type I Diabetes

•Autoimmune destruction of beta cells •Low levels of insulin (and C-peptide) •Inhibition of anabolic processes: glycolysis, glycogenesis, fatty acid synthesis, and protein synthesis •Stimulation of catabolic processes: gluconeogenesis, glycogenolysis, and lipolysis

Presentation of Pheochromocytoma

•Episodic headache, tachycardia, diaphoresis •Paroxysmal vs essential hypertension •Rarely, episodic hypotension and rapidly cycling hypotension/hypertension in epinephrine secreting •Others - Pallor - Postural hypotension (intravascular depletion) - Visual blurring - Dilated cardiomyopathy - Insulin resistance, IGT, type 2 DM

Hypoglycemia workup?

•Fasting plasma glucose •C-peptide •Insulin •Pro-insulin •Abs to insulin or receptor •24 hour observed fast •Whipple's triad

Cushing's syndrome - making the diagnosis?

•First confirm diagnosis of hypercortisolism - Elevated 24-hr urine cortisol - Elevated 11pm or midnight salivary or plasma cortisol - Low dose dexamethasone suppression test showing impaired suppression of cortisol secretion •Then determine the cause of the hypercortisolism. Measure plasma ACTH, - If high, then ACTH-dependent form - If low, then ACTH-independent form, then you order adrenal imaging •If elevated ACTH, need to determine the source by high dose dexamethasone suppression test - If suppression, then pituitary adenoma - If not suppression, then ectopic secretion •Then can use imaging to CONFIRM what you already know

Complications of Cushing's

•Hypertension: Plasma volume expansion, upregulation of adrenergic receptors •Osteoporosis: Stimulates osteoclasts, inhibits osteoblasts •Avascular necrosis •Diabetes: Inhibits peripheral utilization, stimulates hepatic gluconeogenesis, stimulates lipolysis, stimulates muscle breakdown •Immunosuppression: Inhibits phospholipase A2 production of arachidonic acid

Which is faster, cyclooxygenase or lipoxygenase?

Cyclooxygenase will be faster

Why does BUN increase in Addison's?

Due to volume contraction

Myxedema?

Edema of subcutaneous tissue with increased level of IFS proteoglycans. Patients have droopy eyes, increased fatigability, and dull and expressionless facies

Does exophthalmia affect one or both?

Either or both

What is the MOA of pamidronate and etidronate?

Inhibit osteoclastic activity reduce bone resorption and hydroxyapatite formation.

ANTITHYROID MEDICATIONS PTU and Methimazole

Inhibit thyroxine synthesis and release Side effects: - Goiter (hypothyroidism) - Agranulocytosis - Cross placenta Do not affect thyroglobulins that are already stored in the Thyroid. Oral administration may require several days for effects to show up (i.e. until thyroglobulin resources are exhausted. Antithyroid Medications in Pregnancy - Iodine can cross the placenta and will be concentrated in the fetal thyroid - The fetal thyroid does not concentrate iodine during the first 12 weeks of gestation; thereafter iodine uptake increases progressively until birth. - there is only a very low transfer of maternal TSH, T4, and T3 across the placenta. However, iodine, thioamides, and TRH can cross the placenta very readily - Use lower dose of thioamides or partial thyroidectomy (PTU > Methimazole) - Propylthiouracil is preferable over methimazole in pregnancy. - In pregnancy: Use lower dose or perform partial thyroidectomy (during the 2nd trimester) - Do not give 131-I in pregnancy or to nursing mothers. It crosses placenta and may destroy fetal thyroid. - Radioiodine therapy is most suitable for adults over 21 and for patients who do not respond to antithyroids or to subtotal thyroidectomy - Thioamides, PTU and methimazole inhibit thyroxine synthesis and release - SEs: Goiter and agranulocytosis; both can cross the placenta

Prostacyclin is made from what?

Intact endothelium cells

What is Leptin?

Leptin, a peptide hormone that is produced by fat cells and plays a role in body weight regulation by acting on the hypothalamus to suppress appetite and burn fat stored in adipose tissue. Cortisol suppresses leptin and/or down-regulates leptin receptors. Ob(Lep) gene (Ob for obese and Lep for leptin) is located on Chromosome 7 in humans (Lepto=Greek=Thin)

Top drugs that reduce thyroid function

Lithium Amiodarone (hypo or hyper) Phenytoin and carbamazepine Interferons (for hepatitis or MS)

Type 1 is in where?

Liver, pituitary, gonads, brain, adipose, adrenals

A30-year-old married female is being evaluated for inability to conceive, galactorrhea, amenorrhea and menstrual irregularities of long-duration. She has normal blood pressure, and no palpable masses in her breasts. Lab analysis confirms decreased LH and estradiol levels. This patient is most likely suffering from which of the following conditions?

Prolactinoma

What is the most common pituitary tumor?

Prolactinoma

Why do COX2 inhibitors cause MI, thrombosis, and stroke?

Prostacyclin is the dominant prostanoid produced by endothelial cells. It causes local smooth muscle relaxation and vasodilation and interacts with platelet IP receptors and inhibits aggregation. Platelets have only COX-1 which converts arachidonic acid to vasoconstrictive and platelet aggregating eicosanoid thromboxane A-2 Selective inhibition of COX-2 would result in reduction in endothelial production of prostacyclin but leaves platelet production of TXA2 intact This imbalance increases risk of CVA

What is the cause of easy bruising in Cushing's?

Protein degradation, loss of connective and soft tissue proteins such as collagen as a result of potassium loss and diabetic-like sx

What causes pseudoprecocious puberty?

Pseudoprecocious puberty may be caused by exogenous hormones, adrenal tumors, congenital adrenal hyperplasia (e.g., 21-hydroxylase deficiency), hormone-secreting tumors, or McCune-Albright syndrome in females (ovarian cysts, pseudoprecocious puberty, polyostotic fibrous dysplasia of bone, and café au lait spots)

Thyroid Storm

Sudden exacerbation of hyperthyroid signs and symptoms, fever and tachycardia. Precipitated by illness, trauma, surgery, and childbirth, molar pregnancy May progress to coma and death.

How is hypothyroidism treated?

With T4 or thyroxine. T3 should not be used. In older patients, it is important to "start low and go slow" because overtreatment can be dangerous.

A patient presents with exophthalmia, is it possible that she have hypothyroidism?

YES! Hashimoto's may also present with exophthalmos.

Arachnidonic acid => ?

(Lipoxygenase) Leukotrienes (Increase bronchial tone)

Membrane Phospholipid => ?

(Phosphlipase A2) => Arachidonic acid ( stopped by corticosteroids )

Opposing Vascular Effects TxA2? (platelet)

(Platelet) Increases platelet aggregation and vasoconstricts

Treatment of Hypocalcemia

- Calcium salts - Calcium chloride - Calcium gluconate - Calcium carbonate - Vit D For calcium less than 7.5

Most common cause of hypocalcemia

- Chronic renal failure - Hypoparathyroidism - Vit D deficiency - Malabsorption

Acromegaly Presentation?

- Debilitating headache - Course facial features •Frontal bossing •Enlarged jaw (macrognathia) •Enlarged nose, tongue, and lips •Enlarged hands and feet •Enlarged thyroid, heart, liver, and kidneys •Overt diabetes in 10-15%, 50% with IGT - Other: Fatigue, visual disturbances, sleep apnea, amenorrhea, impotence

Physiology of Prolactin

- During pregnancy •Breast maturation and lactogenesis •Lactogenesis inhibited by high estrogen and progesterone - Post pregnancy •Stimulates lactation in response to suckling •Natural contraceptive for breastfeeding women - Role in men •Limited role •Impotence and loss of libido, decreased testosterone

What are some major neurological sx of hypocalcemia?

- Hallmark: neuromuscular irritability - Numbness and tingling of fingers and toes - Muscle ramping - Carbopedal spasm (flexor spasm of the arms and extensor spasm of the legs) - Tremor and Chorea - Seizures

1. A45-year-old woman with Cushing's disease. 2.A 35-year-old woman with Addison's disease. 3.A postpartum woman who is presented with Sheehan syndrome after delivery. 4.A 12-year-old girl diagnosed with 17-hydroxylase deficiency. 5.A 149 two-month child diagnosed with hydroxysteroid dehydrogenase deficiency 6.A 4 year-old boy with precocious puberty and adrenal hyperplasia 7.A patient who is diagnosed with oat cell carcinoma of the lung

1. High levels of urinary 17-OH CS (this is more common with exogenous cortisol or adenoma of zona fasciculata) High levels of ketosteroids in urine 2. Low levels of both 17-OHCS and ketosteroids 3. Low levels of both 17-OHCS and ketosteroids 4. Low levels of both 17-OHCS and ketosteroids 5. Low levels of both 17-OHCS and ketosteroids 6. High levels of urinary ketosteroids and Low levels of urinary 17-OHCS 7. High levels of both 17-OHCS and ketosteroids in urine

Primary hyper-aldo renin is what? Secondary hyper-aldo renin is what?

1. Low 2. High

Tx of Salicylate poisoning

1.Gastric lavage 2.Maintenance of respiration 3.Alkalinization of urine

Salicylate Poisoning

10 - analgesic, antipyretic and antiplatelet functions 50 - anti-inflammatory 100 - tinnitus and central hyperventilation 150 - metabolic acidosis and dehydration 150+ - vasomotor collapse, renal and respiratory failure, coma and severe dehydration

10% rule of pheo?

10% Bilateral; 10% Familial; 10% Malignant; 10% Calcify; 10% Extra-adrenal, and 10% in Children

PHEOCHROMOCYTOMAS

10% Malignant Highest level of catecholamines Malignant HTN Clonidine fails to suppress

NEUROFIBROMATOSIS

10% malignant Autosomal dominant 100% inherited Most common Meningioma (II) Astrocytoma (II) Pheochromocytoma Cafe-au-lait skin spots Pigmented iris Hamartomas (I)

NEUROBLASTOMAS

100% malignant children under 5 Abd mass

Which two are hypertensive?

11 and 17 deficiency

Progesterone => 17-hydroxylase =>?

17-OH progesterone

At what gestational time do fetal thyroid produces thyroxine?

18-20 weeks of gestation (almost after halfway through pregnancy!) Note: If maternal TSI or TRH crosses placenta after this age they cause hyperthyroid symptoms in the fetus

Klinefelter's

47, XXY karyotype •Testicular dysgenesis •Male hypogonadism •Increased urinary 17-ketosteroid •Increased FSH •Gynecomastia, and lack of spermatogenesis •Impotence, and •Mild mental retardation

The difference between Aspirin and salicylic acid is that Aspirin is Acetylsalicylic acid. What is the major difference between aspirin and other NSAIDs, including salicylic acid, with regard to the inhibition of the COX enzyme?

Aspirin irreversibly acetylates COX. All others do so reversibly.

ACTH effects on the adrenals

ACTH acts on the adrenal cortex and causes hypertrophy of zona fasciculata and zona reticularis—sites of cortisol and androgen precursors synthesis. ACTH up-regulates ACTH receptors Increases cholesterol uptake via HDL/LDL receptors (increases adrenal free cytoplasmic cholesterol mainly in the adrenal glands). Increases desmolase activity. Hence, increases cholesterol conversion to pregnenolone. Increases cytochrome P450 enzymes and causes an increased conversion of pregnenolone to cortisol and aldosterone

DDx of Cushing's

ACTH-dependent Cushing's syndrome •Cushing's disease •Ectopic ACTH secretion •Ectopic CRH secretion ACTH-independent Cushing's syndrome •Adrenal adenoma •Adrenal carcinoma •Micronodular hyperplasia •Macronodular hyperplasia Pseudo-Cushing's syndrome •Major depressive disorder •Alcoholism

Cushing's Presentation

ACTH-independent forms •Central obesity that spares the extremities (extremity wasting may even occur) •Rounded face ("moon facies") •Abdominal striae •Dorsocervical fat pad ("buffalo hump") •Hyperglycemia •Osteoporosis •Hypertension •Muscle wasting ACTH-dependent forms •Hirsutism •Oligomenorrhea •Acne •Deepening of the voice

In diagnosing GH-producing adenomas, there is a key relationship between GH and glucose. What is this key relationship?

Administration of glucose will lower GH level in normal individuals, but it fails to do so in GH-secreting adenomatous patients

Conditions decreasing urinary 17-OHC

Adrenal congenital hyperplasia Addison's Hypopituitarism

Three steroids with noticeable mineral effect

Aldosterone Fludrocortisone Deoxycorticosterone

Potassium in Primary Hyperaldosteronism

Aldosterone increases sodium reabsorption, and excretion of K+ and H+ in the distal renal tubules. K+ and H+ loss cause metabolic alkalosis Hypokalemia causes muscle weakness, paresthesias and tetany

What is the reason for osteoporosis in Cushing's?

Cortisol slows down osteoclastic removal of old bone and as a result slows down new bone formation or osteoblastic functions!

Adrenal adenoma and adenocarcinoma

Cortisol up is primary cause ACTH decreased No effect of dexamethasone effect

COX-2?

Bad COX Inflammatory and immune cells (neutrophil, macrophage and mast cells) Induced by pro-inflammatory cytokines (interleukin-1), growth factor, and endotoxins Classic NSAIDs can inhibit both COX-1 and COX-2.

How is precocious puberty treated?

Because premature puberty causes premature fusion of growth plates in the bone and can cause serious social problems for affected children, treatment is indicated. Treatment of any underlying disorders is indicated for pseudoprecocious puberty. For true idiopathic precocious puberty, treatment with long-acting GnRH agonists is indicated to suppress the pituitary-hypothalamic axis and to delay the onset of puberty until an appropriate age.

What are the three well-known COX-2 inhibitors?

Cox-2 Inhibitors: Celebrex (Celecoxib), Vioxx (Rofecoxib) and Bextra (valdecoxib)

Cushing's and Addison's disease have opposing effects on the level of eosinophils. Which one causes eosinophilia and which one causes eosinopenia

Cushing's (hypocortisolism) depresses immune functions and causes eosinopenia. Addison (lack of steroids) does the opposite (eosinophilia

Pituitary

Cushing's disease corticotropic adenomas High ACTH Bilateral adrenal hypertrophy

Glucocorticoid (Cortisol) Side Effect

Hypokalemic alkalosis Glycosuria Increased susceptibility to infection Peptic ulcers (synthesis of mucosa of stomach require PGs) Myopathy (more common with long term dexa and triamcinolone) Osteoporosis Cushing's syndrome and hypertension ACTH suppression

Why is selective inhibition of COX2 preferable over non-selective inhibition of COX 1 & 2?

COX 2 is confined to the inflamed tissues

What hormone is helpful in tx of hypercalcemia and in tx of Paget's?

Calcitonin

Vit D Preps?

Calcitriol (active form): rapid function Ergocalciferol Calciferol

Levels of all minerals increase in serum in renal disease with the exception of what mineral and why?

Calcium decreases! Due to lack of calcitriol as a result of alpha-hydroxylase deficiency and hyperphosphatemia that further inhibits activation of vitamin D and the fact that phosphate binds free calcium and drops free blood calcium levels.

chimpanzees for causes of hypercalcemia?

Cancer Hyper (parathyroidism, vitaminosis A and D, HCTZ) Immobilization Multiple myeloma; milk-alkali Paget's Dz Addison's; Acromegaly; Androgens Nephrogenic Zollinger-Ellison Estrogen treatment Endocrine Sarcoidosis

Rickets and Osteomalacia

Causes - Vitamin D deficiency - Decreased Ca++and phosphate reabsorption from GI causes decreased hydroxyapatite/collagen ratio in bone (i.e. reduced amount of mineral per matrix). Hence, bones soften. Rickets (children); Osteomalacia (adults) Characteristics: Lack of proper bone mineralization and mechanical stress of supporting structures leads to characteristic outward bow of the bones. Diagnosis: X-ray and diet history Treatment: Vitamin D and Calcium

SUMMARY OF MAJOR PGs PGF

Causes Uterine Contraction and Induction of Labor Causes bronchoconstriction

Therapeutic Uses Of Desmopressin

Central diabetes Insipidus Nocturnal diuresis Note: Desmopressin has greater antidiuretic but less vasopressin effect compared to ADH

What causes SIADH?

Central nervous system causes: Stroke, hemorrhage, infection, trauma. Medications: Narcotics, oxytocin (in pregnant patients), chlorpropamide, antiepileptic agents. Trauma: Pain is a powerful stimulus for ADH. Watch for the postoperative patient who is receiving fluids (and often narcotics) and has pain. Lung problems: Simple pneumonia or ADH-secreting small cell cancer of the lung.

Functional Adenoma Non-functioning adenoma, colloid nodule, cysts and malignant tumors Local thyroiditis

Increased I-123 uptake Decreased I-123 uptake Increased or decreased I-123 uptake

What is the most profound serum finding in this patient?

Increased alkaline phosphatase

SKELETAL MUSCLE IN FAST

Increased glucose Sparing Decreased glucose import Decreased glycolysis Increased FFA import Increased beta-oxidation Increased amino acid export to live

LIVER IN FED STATE

Increased glycogen synthesis Decreased glycogenolysis and gluconeogenesis Increased glucose import and glycogenesis Decreased amino acid import Decreased urea formation and transport to kidney Increased glycolysis and glycerol synthesis Decreased FFA import from adipose tissue Decreased beta-oxidation and ketogenesis Increased lipogenesis, TG and cholesterol synthesis TG exports to plasma via VLDL

BRAIN IN FAST STATE

Increased ketone body import (derived from liver) No significant change in glucose import

Define diabetes insipidus (DI). What are the two types?

DI is a lack of antidiuretic hormone (ADH), or vasopressin, effect in the body. Patients with DI secrete inappropriately dilute urine because of the lack of ADH effect and may urinate up to 25 L of urine per day, resulting in dehydration and hypernatremia. Such patients die rapidly if they are unable to drink water. Normally, when the body is dehydrated, ADH causes urine to become highly concentrated through retention of free water. In DI, the urine remains dilute even though the serum osmolarity is quite high as a result of dehydration. The two types are central and nephrogenic.

Calcitonin on kidney?

Decrease Calcium reabsorption and increases its excretion

What is Ghrelin?

Ghrelin is a hormone produced by the fundus of the stomach and by the pancreas that stimulates hunger. Ghrelin levels increase before meals and decrease after meals. It is considered the counterpart of the hormone leptin, produced by adipose tissue, which induces satiation when present at higher level

Define hirsutism. What causes it?

Hirsutism is a male hair growth pattern in women or prepubescent children. The most common cause is familial, genetic, or idiopathic hirsutism, but on the USMLE watch for polycystic ovary syndrome(Stein-Leventhal syndrome), Cushing syndrome, and drugs (minoxidil, phenytoin, cyclosporine). These disorders do not produce virilization. If virilization (clitoral enlargement, deepening of the voice, temporal balding) accompanies the hirsutism, an androgen-secreting ovarian tumor (e.g., Sertoli-Leydig cell tumor, arrhenoblastoma) or adrenal source (congenital adrenal hyperplasia, Cushing syndrome, or adrenal tumor) is likely

Hot versus Cold Nodules

Hot Nodules: 85% of benign adenomas. Present with highest accumulation of radio-iodine on scintigraphy. Cold nodules: Hypofunctional or non-functional adenomas. Often malignant.

A 38 yo woman presents with a 2 month history of gradual weight loss, fatigue, and heat intolerance with palpitations. One exam,s eh has BP of 170/70. Her pulse rate is 95. Labs show low TSH and high T4. She is scheduled to receive a FNA biopsy - what will be a better prognosis?

Hot nodule

Short acting glucocorticoids

Hydrocortisone and cortisone Duration of action: 8-12 hours

Dx of hyperparathyroidism?

Hypercalcemia + Hypophosphatemia

A 55-year -year-old male gradually becomes weak and develops diabetes. Autopsy is significant for skin pigmentation, oat cell carcinoma of the lung, and adrenal hyperplasia. Patient lacks truncal obesity, and skin striations. The patient would most likely have suffered from which of the following acid-base imbalances?

Hypokalemic alkalosis due to mineralocorticoid effect Increase ACTH precursors will darken his skin. Ketoconazole will be most helpful for the patient - it's antifungal that inhibits the conversion of cholesterol to subsequent steroids. Desmolase inhibitor Spironolactone is only helpful for too much aldosterone or too much activity within zona glomerulosa.

Cause: Cushing disease

Increase ACTH - primary cause Increase Cortisol - follow High doses suppress ACTH and cortisol

Hyperthyroidism Treatment: AS cause of goiter

Methimazole or PTU => Decreased T3 and T4 => Increased TSH => Follicular stimulation => Goiter

Oat cell carcinoma

Mets early Produces ADH and ACTH (cushing's) Tx: Chemotherapy or radiation

classic case of cushing's

Middle-aged female •Weight gain and increased hair •Truncal Obesity and moon facies •Easy-bruising (diabetes-like effect) •Elevated fasting glucose •Elevated plasma cortisol •Eosinopenia and leukopenia •Adrenocortical hyperplasia (with Cushing's disease and ectopic causes) •Increased androgenic, glucocorticoid and mineralocorticoid effects

Pheochromocytoma

Most common adult tumor of the adrenal medulla Increased production of catecholamines Associated with MEN IIa, b and III Sporadic form (40-60 years of age); more in females. Familial form (younger age of onset) is more common in boys Findings in Pheochromocytoma - Elevated glucose - Increased urinary catecholamines or metabolites (diagnostic) - Sustained or intermittent hypertension - Morphology: Tumors are often vascular, and when fixed with dichromate (i.e. Zenker's) they turn brown-black due to oxidation of catecholamines (Chromaffin cells)

Bronchial tone increase by prostaglandins?

Mucus secretion

2 syndromes of manifestations?

Myxedema: Adults Cretinism: children

What causes hyperparathyroidism?

Ninety percent of primary cases are caused by a parathyroid adenoma, which can usually be confirmed with a nuclear medicine scan. Other causes include parathyroid hyperplasia and parathyroid carcinoma. Secondary cases include low calcium levels (e.g., from renal failure), to which an increase in PTH is a normal physiologic response. Tertiary hyperparathyroidism occurs when PTH has been elevated for too long (secondary to long-standing hypocalcemia) and continues to be over secreted even when calcium is normalized with treatment. Translation: Put all patients with renal failure on calcium supplements to prevent this complication.

BRAIN IN FED STATE

No significant change in glucose import

type 2 is in where?

Only in nephrons primarily in collecting ducts.

Parathyroid hormone

Parathormone (PTH), the parathyroid hormone, is a polypeptide produced by chief cells. - Primary cause of release is decreased plasma calcium level. - Secondary cause of release is increased plasma [PO4] Primary negative releasing factor is increased [Ca++] - Secondary negative releasing factor is decreased [PO4] - Severe [Mg++] drop strongly inhibits PTH secretion Transduction is via G-protein kinase

Diabetic Ketoacidosis

Pathophysiology •Absence of insulin •Increased fatty acids delivered to liver •Beta-oxidation => ketoacids => acidosis => hyperkalemia •Hyperglycemia => osmotic diuresis => dehydration •"Physiologic" hyponatremia Treatment •Aggressive fluid hydration •Insulin with glucose or dextrose •Slow correction or will cause cerebral edema

What is one the top etiologies of excess catecholamines in hypertensive adults?

Pheochromocytoma

What abx drops serum calcium?

Plicamycin Inhibits Vit D or PTH's function

Intermediate acting glucocorticoids

Prednisone, prednisolone, triamcinolone 18 -36 hours

Cholesterol => Desmolase => ?

Pregnenolone

Mechanism of Radioactive Iodine

Rapid uptake by Thyroid Destruction of thyroid tissue Paradox of Iodide: Large doses of iodides (ionic iodine) administration instead of exacerbating thyroid function decrease the function. Iodide Preparations - Lugol Solution - Potassium iodide Do not use 131-I in patients under 21 or in pregnancy

Myxedema coma?

Rare life-threatening condition mostly seen in the elderly women Hypothermic stuporus hypothyroid state due to weakness, hypoglycemia, hypoventilation and hyponatremia that leads to shock and coma. Factors enhancing myxedematous state - Cold exposure, trauma, infection - Aggressive radioactive 131I treatment of Grave's disease (that destroys the thyroid)

What is the underlying mechanism for antipyretic,analgesic and anti-inflammatory functions of Aspirin?

Reduced PG that causes analgesia in nerve endings, antipyretic reaction via hypothalamus and anti-inflammatory reaction at the inflammation sites

PGI

Relaxes vessels Sensitizes to pain Slows stomach acid secretion

Anti-inflammatory Roles of cortisol

Strong inhibition of the entire inflammatory response in the body Decreases PGs, prostacyclin, and leukotriene synthesis by inhibiting phospholipase A2 Prevents transplant rejection Decreases neutrophil recruitment to inflamed tissues Decreases T-cell count, and inhibits production of interleukin-2. (note that IL-2 converts resting T cells to cytotoxic cells and is responsible for development of T immunologic memory) Decreases capillary permeability, and inhibits histamine release from mast cells. Shrinks the lymphoid mass and drops the lymphocytic count. Hence, it is desirable in treatment of lymphomas

Which conditions cause goiter?

Secondary and Tertiary

Bartter's Syndrome

Secondary hyperaldosteronism Seen more in females Renal defect of sodium reabsorption (aldosterone cannot reabsorb sodium). There is renal resistance to angiotensin and aldosterone Urinary sodium wasting (hyponatremia and natriuresis) Increased renin and aldosterone JG hyperplasia Hypokalemic (metabolic) alkalosis Increased renal prostaglandins (in response to lower pressure) Hypotension and lack of peripheral edema

What are the symptoms and signs of hypercalcemia?

Symptoms: "Bones, stones, groans, and psychiatric overtones." In other words: bone resorption with osteomalacia and osteitis fibrosa cystica; kidney stones; abdominal pain secondary to nausea and vomiting, ileus, nephrolithiasis, peptic ulcer disease, constipation, or pancreatitis (all increased with hypercalcemia); and emotional lability, delirium, depression, and/or psychosis. Signs: Shortened QT interval on ECG, weakness, polyuria, bone changes and kidney stones on radiograph, and renal failure

What are the common symptoms and signs of hyperthyroidism?

Symptoms: Nervousness, anxiety, irritability, insomnia, heat intolerance, sweating, palpitations, tremors, weight loss with increased appetite, fatigue, weakness, emotional lability, and diarrhea. Signs: Enlarged thyroid gland, warm skin, thyroid "stare"—lid lag, exophthalmos, proptosis, ophthalmoplegia (Graves disease), pretibial myxedema (Graves disease), tremor, tachycardia, and atrial fibrillation. Check thyroid-stimulating hormone (TSH) when patients come to the hospital with new-onset atrial fibrillation.

What are the symptoms and signs of hypocalcemia?

Symptoms: Paresthesias (the classic pattern is perioral or distal extremities), muscle aches, dementia, depression, and psychosis. Signs: Prolonged QT interval on ECG, tetany, Chvostek sign (tetany elicited by tapping on the facial nerve to cause facial muscle contraction), Trousseau sign (carpopedal spasm caused by inflation of a blood pressure cuff or application of a tourniquet), dementia, depression, psychosis, seizures, and papilledema.

How is Cushing's syndrome diagnosed?

The first test is either a 24-hour measurement of free cortisol in urine (free cortisol levels are abnormally elevated) or a dexamethasone suppression test (cortisol levels are not appropriately suppressed several hours after administration of dexamethasone). Random cortisol level is an inappropriate test because of wide interpatient and intrapatient variations. ACTH is elevated in Cushing's disease but decreased with an adrenal adenoma. If ACTH is increased, a magnetic resonance imaging (MRI) scan of the brain should be obtained to look for a pituitary adenoma. If ACTH is decreased and the patient has no history of taking steroids, an abdominal computed tomography (CT) or MRI scan should be obtained to look for an adrenal tumor. Primary cancer is usually obvious when ectopic ACTH is the cause (e.g., weight loss, hemoptysis with lung mass on chest radiograph in patients with small cell lung cancer). Treatment is based on the cause and usually involves surgery.

What is the relationship between hyponatremia and myxedema coma?

The hyponatremia of myxedema coma is a result of decreased free water clearance. Elevated levels of antidiuretic hormone and/or diminished blood flow to the kidneys are believed to be responsible for the inability to excrete free water

What is the C-peptide level of a type 1 diabetic; low or high or normal?

There is little or no c-peptide in blood of type 1 diabetics because they do not produce endogenous insulin.

Thyroid Carcinomas

Three types: Papillary, follicular and medullary All are malignant but uncommon in USA More in females 2: 1 to 3:1 ratio - Papillary: Most common(75-85% of cases) with good prognosis. Characteristic Orphan Annie eye nuclear inclusions and psammoma bodies. - Follicular: 10-20% of cases. Prevalent in iodine deficient areas. Poor prognosis. - Medullary: Originate from C-cells that are Calcitonin-producing cells. Associated with MEN II-a - hypocalcemia Diagnostic Tumor markers - Serum thyroglobulin - Calcitonin (for medullary type carcinomas—related to MEN IIA)

How is SIADH treated?

Treat with water restriction. Stop intravenous fluids and restrict oral fluid intake. For Step 2 purposes, do not give hypertonic saline unless the patient has active seizures. You may cause brain stem damage or central pontine myelinolysis from too rapid correction of sodium level. Demeclocycline Is sometimes used to treat SIADH if water restriction fails because it induces nephrogenic DI, which allows the patient to eliminate free water.

How is Addison's treated?

Treatment of Addison's disease involves replacing the missing hormones, cortisol (with oral hydrocortisone) and aldosterone (by fludrocortisone)

Define precocious puberty and pseudoprecocious puberty.

True precocious puberty is defined as activation of the hypothalamic-pituitary axis with sexual maturation before the age of 8 years in females and before the age of 9 years in males. In pseudo-precocious puberty, secondary sex characteristics develop prematurely because of high circulating levels of androgen or estrogen.

Adrenal

Unilateral adrenal adenoma high cortisol; low ACTH Atrophy of the non-adenomatous adrenal gland

Dx of Cushing's DISEASE

Using: Dexamethasone suppression test Dexamethasone is 20 times more potent than naturally occurring cortisol levels. In endogenous and exogenous Cushing's dexamethasone will not suppress the plasma and urine cortisol levels However, very high levels of dexamethasone will suppress ACTH output in Cushing's disease (due to pituitary adenoma)

Zona Glomerulosa makes what?

Zona Glomerulosa Makes the hormones that act on the glomeruli—e.g. aldosterone

STEROIDS

Zona glomerulosa lacks 17 alpha-hydroxylase, and it is the only zone that contains aldosterone synthase Zona glomerulosa has no enzymatic pathway to synthesize cortisol, DHEA (dehydroepiandrosterone) and androstenedione Male and female gonads lack 11 and 21 alpha-hydroxylase. Thus, gonads cannot produce corticosterone, cortisol or aldosterone. DHEA and androstenedione can be converted to testosterone in gonads. Adrenal testosterone synthesis is insignificant Gonads only possess enzymes that are required for androgens (e.g. testosterone) and estrogen synthesis

Phenylbutazone

•Powerful anti-inflammatory, weak analgesic •Not a first line drug •Short-term gout therapy •Adverse effect: aplastic anemia and agranulocytosis (most likely)

Does aldosterone-secreting adenoma in Conn's have inhibitory effect on ACTH (or anterior pituitary)?

aldosterone secreting adenomas have no effect on the anterior pituitary. Also, as opposed to adenomas of the zona fasciculata, the adenomas zona glomerulosa have no effect (indirectly via pituitary) on the contralateral adrenal

Acromegaly Anabolic actions

•Promotes tissue growth, particularly of musculoskeletal system •Stimulates liver to secrete IGF-1 •Pathology - Gigantism - Acromegaly - Pituitary dwarfism

Complications?

•Small vessels disease - Retinopathy - Neuropathy - Nephropathy •Large vessel disease - Coronary artery disease - Peripheral vascular disease

The Ps that decrease T3RU and increase TBG?

pregnancy/OCPs Hepatitis (and biliary cirrhosis) Pills/drugs (tamoxifen and perphenazine) Porphyria (acute intermittent) Progesterone

What is von Hippel-Lindau disease?

von Hippel-Lindau disease is due defective VHL gene located on chromosome 3 and it is a tumor suppressor gene that regulates the protein elongin. Patients are presented with hemangioblastomas in the cerebellum and retina

Acetaminophen (and Phenacetin)

•Absorption orally; Elimination renally via glucuronidation •Analgesic and antipyretic; inhibit PG synthesis in the CNS •Hepatotoxicity (necrosis); antidote is N-acetylcysteine •No anti-inflammatory action. Less effect on peripheral COX; No or less bleed

17-OH => 21 hydroxylase?

Deoxycortisol

Deoxycortisol => 11-hydroxylase ?

Cortisol

After tapping the facial nerve at anterior aspect of the ear a patient is presented with facial muscle spasm. What is the problem with this patient?

Hypocalcemia - Chvostek's sign

What type of hypercortisolism is suppressed by high dose of dexamethasone?

Pituitary Cushing's (disease) is suppressed by high doses of dexamethasone

METABOLIC EFFECTS OF CORTISOL

Prepares body for stress by metabolizing fuels Catabolism of muscle protein to provide substrate for liver gluconeogenesis. Increases plasma glucose

CRH via cAMP stimulates what?

Synthesis of POMC which splits into ACTH, MSH, and endorphin

Phenylpropionic Acid

•Ibuprofen, Naproxen, Ketoprofen •Analgesic, antipyretic and anti-inflammatory; lower toxicity than above two classes

Indoleacetic Acids

•Indomethacin and etodolac •Analgesic and anti-inflammatory; More potent anti-inflammatory than aspirin •Indication: gouty arthritis, ankylosing spondylitis and osteoarthritis •Adverse Effects: GI hemorrhage, diarrhea, and neutropenia

Diclofenac

•Long-term treatment of rheumatoid arthritis and ankylosing spondylitis

How do remember Klinefelter's?

•No man can feel a woman better than Kline. •Kline has a complete XX set of chromosomes in addition to a Y. •He has feminine breasts. •His bodily hair distribution looks like a woman. •He has tinny gonads, and doesn't do spermatogenesis. •That's why no man can feel her better than Kline!

Oxicam

•Piroxicam (anti inflammatory) •Interferes with renal excretion of lithium

Same case scenario: Is this patient more likely a man or woman?

Man

A 72 year-old man with long history of RA admitted to ICU with pneumonia. He is hypotensive despite fluids and pressors. Labs show plasma sodium concentration of 124, and potassium of 5.3.Laboratory workup reveals hyponatremia, hyperkalemia, and hypoglycemia. Further workup reveals elevated AM levels of ACTH and low cortisol. Injection of ACTH elicits only a blunted increase in plasma cortisol. He is started on routine hydrocortisone and fludrocortisone.

Dx is Addison's

Describe the laboratory findings in hypothyroidism.

Elevated TSH (unless caused by secondary causes), decreased T3 and T4, antithyroid and anti microsomal antibodies (if caused by Hashimoto's thyroiditis), hypercholesterolemia, and anemia (which may be because of chronic disease or coexisting pernicious anemia).

After 72 hours of fasting a seriously hypoglycemic patient fails to suppress his insulin level. This would strongly suggest what pathological condition?

Failure to suppress insulin secretion a hypoglycemic patient after 48-72 hours of fasting indicates presence of insulin-producing tumors

Primary Hyperparathyroidism

Findings •Kidney stones and nephrocalcinosis •Osteitis fibrosa cystica •Osteoporosis, osteomalacia and arthritis •Constipation •Keratopathy •Psychosis and delirium Diagnosis: High plasma calcium and elevated urinary cAMP. Most common cause: Sporadic, single parathyroid adenoma Less common causes: Hyperplasia of all parathyroid glands, carcinomas, adenomas that occur in more than one gland, MEN 1 and MEN 2A

Why is free T4 (or free T4 index) better than total T4 for measuring thyroid hormone activity?

Free T4 (free T4 index) measures the active form of thyroid hormone. Many conditions cause a change in the amount of thyroid-binding globulin (TBG), thus changing total T4 levels in the absence of hypothyroidism or hyperthyroidism. Common examples include pregnancy, estrogen therapy, and oral contraceptive pills, all of which increase TBG. Nephrotic syndrome, cirrhosis, and corticosteroid treatment all decrease TBG. T3 resin uptake is an older test that is not likely to appear on Step 2, but if you are asked, it should rise or fall in the same way as free T4. Although an oversimplification, this principle should serve you well on the examination.

What diagnostic test can reveal whether DI is central or nephrogenic? How are these conditions treated?

Give the patient a dose of ADH, and measure urine osmolarity. If central DI is the cause, urine osmolarity increases with ADH challenge. In nephrogenic DI, the urine remains inappropriately dilute after the patient is given ADH. Treatment for central DI is ADH replacement (given orally or as a nasal spray). Treatment for nephrogenic DI involves stopping any offending drug and giving a thiazide diuretic; ADH does not help. Although giving a diuretic to a patient with DI seems counterintuitive, it has the paradoxical effect of decreasing urine output.

How is hypoadrenalism diagnosed?

An ACTH stimulation test can be done. Plasma cortisol is measured, ACTH is administered, and cortisol is measured again in 1 hour. The cortisol level should rise appropriately, usually to 18 μg/dL or doubling of the baseline, depending on the baseline value. An inappropriate response to ACTH indicates hypoadrenalism. Do not withhold treatment to make a diagnosis if the patient is crashing.

What is sick euthyroid syndrome?

Any patient with any illness may have temporary derangements in thyroid function tests that resemble hypothyroidism. TSH ranges from normal to mildly elevated, and serum T4 ranges from normal to mildly decreased. Clinical circumstances and physical findings are the best guides to whether the patient has true hypothyroidism. In patients with sick euthyroid syndrome, simply treat the underlying illness. If the diagnosis is in doubt, repeat the thyroid tests after the patient recovers (preferred) or give an empirical dose of levothyroxine (if the patient does not respond to treatment of the underlying illness).

What is the definitive treatment for primary hyperparathyroidism?

Definitive treatment of hyperparathyroidism is removal of the primary cause that often means surgical removal of the adenoma.

What is the definitive treatment for insulinoma?

Definitive treatment of insulinoma is by surgical removal of the tumor.

Differential Diagnosis of Hyperthyroidism

Hyperthyroidism with high radioiodine uptake •Grave's disease •"Hashimoto-toxicosis" •Toxic adenoma •Toxic multinodular goiter •Iodine-induced hyperthyroidism •Trophoblastic dz and germ cell tumors •TSH mediated hyperthyroidism Hyperthyroidism with low radioiodine uptake •Thyroiditis - Painful: subacute (de Quervain's), radiation-induced, trauma-induced, infection-induced - Painless: subacute lymphocytic, Hashimoto's, Amiodarone-induced •Exogenous and ectopic hyperthyroidism - Factitious ingestion - Struma ovarii

What causes virilization in children?

In female neonates, congenital adrenal hyperplasia is a likely cause of virilization. The classic example is a female infant born with ambiguous genitalia. However, the patient may also be a male child with precocious puberty. At least 90% of cases are because of 21-hydroxylase deficiency. Because 21-hydroxylase is involved in the production of both aldosterone and cortisol, children develop signs of hypoadrenalism, with salt-wasting, hypotension, hyperkalemia, hyponatremia, hypoglycemia, acidosis, and nausea and vomiting. Abnormally high levels of serum 17-hydroxyprogesterone or urinary 17-ketosteroids (dehydroepiandrosterone [DHEA], DHEA sulfate, and androsterone), along with decreased free cortisol in the serum, clinch the diagnosis. Give corticosteroids to prevent death. In older children, worry about a testosterone-secreting gonadal neoplasm

What is Nesidioblastosis?

It is a term that describes hyperinsulinemic hypoglycemia due to excessive beta cell function in the presence of histological islet changes such as islet dysplasia and proliferation. The term for quite a long time was used to describe congenital hyperinsulinism. However, the most modern usage of the term pertains to adult acquired hyperinsulinism associated with hyperplasia of beta cells, in particular after gastrointestinal surgery.

What is Langerhans' histiocytosis?

Langerhans Cell Histiocytosis (LCH) is a rare disease associated with clonal proliferation of Langerhans cells. Major organs affected are: (1) bones, wherein osteolytic lesions lead to pathological fractures; (2) skin, wherein the patients are presented with rash and erythematous lesions; (3) bone marrow, that leads to pancytopenia and anemia; and (4) endocrine glands and in particular hypothalamic-pituitary axis


Related study sets

4.5-4.6: Free Fall: How Fast & How Far

View Set

1.12.M - Lesson: Topic 8: Virginia and the Civil War Review

View Set

Transcultural Healthcare- Chapter 8 Vocabulary, Workbook, Notes and Learning Objectives

View Set

Science 3-3 Producing Visible Light

View Set